66 Contemporary Issues in Diagnostic Hemostasis and

Roger Riley MD

2011 Annual Meeting – Las Vegas, NV

AMERICAN SOCIETY FOR CLINICAL PATHOLOGY 33 W. Monroe, Ste. 1600 Chicago, IL 60603

66 Contemporary Issues in Diagnostic Hemostasis and Thrombosis

This interactive session will utilize a series of five relevant case studies to explore several contemporary, clinically relevant problems in hemostasis and thrombosis, including anticoagulant monitoring and pharmogenomics, function evaluation, the detection of antiphospholipid and heparin/PF4 antibodies, and the evaluation of patients with suspected hereditary thromboembolic disease. Each case is focused on an important clinical or laboratory problem encountered by pathologists, medical technologists, and hemostasis laboratories. Following a presentation of the basic clinical and laboratory features of each case, session participants will be invited to discuss practical, cost-effective means for further assessment and resolution of the problem and related clinical and laboratory problems. The session will emphasize the integration of modern techniques and discoveries with conventional clot-based assays.

• Interpret important hemostatic laboratory assays and provide recommendations for further patient evaluation or monitoring. • Recognize technical pitfalls the diagnostic limitations of common hemostatic assays, and develop alternative strategies for laboratory evaluation. • Understand emerging and expected future developments in hemostatic testing, such as the increasing role of molecular analysis and point of care testing.

FACULTY:

Roger Riley MD

Practicing Pathologists Blood Banking, Transfusion Medicine Blood Banking, Transfusion Medicine 2.0 CME/CMLE Credits

Accreditation Statement: The American Society for Clinical Pathology (ASCP) is accredited by the Accreditation Council for Continuing Medical Education to provide continuing medical education (CME) for physicians. This activity has been planned and implemented in accordance with the Essential Areas and Policies of the Accreditation Council for Continuing Medical Education (ACCME).

Credit Designation: The ASCP designates this enduring material for a maximum of 2 AMA PRA Category 1 Credits™. Physicians should only claim credit commensurate with the extent of their participation in the activity. ASCP continuing education activities are accepted by California, Florida, and many other states for relicensure of clinical laboratory personnel. ASCP designates these activities for the indicated number of Continuing Medical Laboratory Education (CMLE) credit hours. ASCP CMLE credit hours are acceptable to meet the continuing education requirements for the ASCP Board of Registry Certification Maintenance Program. All ASCP CMLE programs are conducted at intermediate to advanced levels of learning. Continuing medical education (CME) activities offered by ASCP are acceptable for the American Board of Pathology’s Maintenance of Certification Program. Contemporary Issues in Diagnostic Hemostasis and Thrombosis Disclosure Statement

I declare that neither my immediate family members nor I have a financial in- terest in or other relationship with a manufacturer/provider of any commer- cial product/service presented/discussed in the workshop.

Roger S. Riley, M.D., Ph.D.

September 2, 2011

Roger S. Riley, M.D., Ph.D. Professor of Pathology Director, Hemostasis Laboratory Co-Director, Hematology Laboratory MCV Campus, VCU School of Medicine 665-E CSC Building, 403 North 13th Street Richmond, VA 23298 Telephone: 804-828-0338 Email: [email protected] The Case of the Art History Major with Chronic

Clinical History: It was a dark and stormy fall eve- ning in Mason City, MO. Suzanne Cezanne was a 21 year-old art historian and assistant curator at the Art Museum of the Midwest in Mason City, MO. She was working alone in the Edgar Allen Poe exhibit, on loan from the Edgar Allen Poe Museum in Richmond, VA, and scheduled to open the next day. She had worked on the exhibit since early morning and hadn’t had time to eat lunch or dinner. There was great interest in the exhibit, since one of the founders of Mason City, Judge Nathaniel B. Tucker, was a good friend of Poe. As she was adjusting an old smokey, floor- length mirror, the sharp features and piercing eyes of Edgar Allen Poe were suddenly staring at her. She gasped and almost fainted. For a moment thought Poe’s ghost was in the room, but then realized that it was only a reflection from his picture on the wall be- hind her.

Suzanne knew that Poe had tuberculosis and other medical problems, and she wondered if he also had thrombocytopenia, a problem she had dealt with all of her life. Maybe Dr. Marcus Carrington, Director of the Midwest Hemostasis Institute, could do some- thing for her when she had her first appointment there next week. Case #1 Case #1 - Clinical History 3

Dr. Carrington found that Suzanne had a history of one occasion when her platelet count was 8 x 109/L, severe, chronic thrombocytopenia first diagnosed at she was given several doses of . Her plate- two years of age. Her platelet counts usually varied let count rose to 48 x 109/L one hour later, but it was from 5 x 109/L to 20 x 109/L Other than increased only 18 x 109/L when she returned to the clinic the bruising, occasional episodes of epistaxis, and following week. Suzanne was given a diagnosis of heavy menstrual periods, she had not seriously suf- refractory chronic immune thrombocytopenic pur- fered from this problem. Multiple and extensive pura. evaluations for autoimmune, metabolic, and hema- tologic diseases at other medical centers had been Physical Examination: A physical examination by negative negative. These workups had shown nor- Dr. Carrington revealed an alert, oriented young mal serum B12 and folate levels, a normal erythro- woman with the following vital signs: BP - 115/79, cyte sedimentation rate, negative assays antinuclear pulse - 72/min, temperature - 98.1oF, and weight - antibody and rheumatoid antibodies, a negative di- 57.3 kg. Dermatologic examination showed some rect Coombs test, and normal immunoglobulin lev- bruising at different stages on her upper and lower els. Serologic assays for EBV, CMV, HIV, and other extremities. The cardiovascular and respiratory ex- infectious diseases were also negative, and a aminations were unremarkable. Her abdomen was splenic ultrasound at age 19 showed the spleen to soft and tender and no organomegaly was found. be at the upper limits of normal for her age. Bone There were no palpable cervical, supraclavicular, or marrow evaluations at ages 19 and 20 revealed inguinal lymph nodes. Her sclera were clear and adequate megakaryocytes with normal morphologic anicteric. features, and cytogenetic analysis of the bone mar- row aspirates had showed a normal female karyo- Laboratory Data: Suzanne’s blood was drawn for type. Since Suzanne’s parents perished at an early laboratory studies. A comprehensive metabolic age in an automobile accident, she grew up in a fos- panel, including liver and renal function tests, was ter home and did not have any knowledge of her within normal limits. Blood counts and indices were family medical history. as follows: i

Suzanne had received high-dose steroids, intrave- nous immunoglobulin, and WinRho injections at dif- ferent times with no effect on her platelet counts. At the age of 20 years, she was also given four treat- ments of Rituxan, but her platelet count still re- mained in the range of 5 x 109/L to 20 x 109/L. On

15 6 40 60 150 40 50 50 400 15

5 40 40 13 30 50 125 30 300 10 4 30 30 11 % fL fL

3 20 % 40 100 20 200 pg g/L g/dL 10e9/L 10e9/L 20 20 10e9/L 9 5 2 10 30 75 10 100 10 10 7 1

0 0 0 20 50 0 0 0 0 5 WBC RBC Hgb Hct MCV MCH MCHC RDW PLTs MPV

Fig. 1. Suzanne’s CBC data. Red dot represents patient value, reference ranges are indicated by gray bars.

Contemporary Issues in Diagnostic Hemostasis and Thrombosis ASCP Annual Meeting, October 2011 Case #1 - Discussion 4

Learning Objectives: (1) Understand the etiology of chronic thrombocytopenia, (2) Understand the classifi- cation and clinical features of familial thrombocytopenias, (3) Understand the significance of peripheral blood smear examination and other laboratory assays in the diagnosis of familial thrombocytopenia, (4) Un- derstand the pathogenesis of the MYH-9 related diseases, (5) Understand the treatment options for patients with chronic thrombocytopenia.

Questions:

1. Discuss the etiology of chronic thrombocytopenia. What possible causes of Suzanne’s thrombocytopenia have not been evaluated? In view of her low platelet counts, why have Suzanne’s clinical symptoms been relatively mild?

The general causes of thrombocytopenia include: (1) Artifactual thrombocytopenia due to anticoagulant- dependent platelet agglutinins or other causes, (2) Immune or non-immune destruction of platelets in the circulation, (3) Sequestration of platelets by the spleen, and (4) Failure of platelet production by the bone marrow.

Artifactual thrombocytopenia should be excluded in the evaluation of every new patient with thrombocy- topenia. This can easily be done by reviewing the peripheral blood smear for evidence of platelet clump- ing and satellitism around neutrophils. Other factors that may produce a falsely low platelet count when automated particle counters are used include paraproteins, prior exposure of blood to dialysis mem- branes, giant platelets, and lipemia.1

The history of chronic thrombocytopenia beginning at an early age suggests a possible hereditary etiol- ogy, although chronic immune thrombocytopenia cannot be excluded. Although Suzanne’s family mem- bers are not available for evaluation, repeated evaluation has no revealed evidence of an autoimmune, infectious, or other reason for an acquired thrombocytopenia. Suzanne’s relatively mild sympto- matology in the presence of severe thrombocytopenia suggests intact thrombopoiesis and the continuing production of young, physiologically active platelets. This phenomenon is often seen in patients with im- mune thrombocytopenia.

Dr. Carrington performs a bone marrow examination and peripheral blood smear (film) examination. Images from the peripheral blood smear are shown below.

Contemporary Issues in Diagnostic Hemostasis and Thrombosis ASCP Annual Meeting, October 2011 Case #1 - Discussion 5

2. Does the peripheral blood smear provide any clues about the etiology of Suzanne’s thrombocy- topenia? How would you classify her disease from the peripheral blood smear findings?

Peripheral blood smear examination confirms the presence of severe thrombocytopenia. Most of the plate- lets are large and show normal granulation. The elevated mean platelet volume (MVP) of 10.3 fL (normal range 6.3 - 9.1) provides additional evidence to support this conclusion. Careful examination of the periph- eral blood smear also provides another important clue to the etiology of Suzanne’s disease process; many of the neutrophils contain pale, bluish-purple Dohle-like cytoplasmic inclusions, usually located at the pe- riphery of the cell.

There are many congenital platelet diseases (Table I). In general, these diseases and very rare, and each is characterized by a different constellation of clinical and laboratory findings.2-6 The platelet size, presence or absence of thrombocytopenia, platelet size, and other hematologic and non-hematologic clinical features are important diagnostic considerations.

Table I Clinical and Laboratory Features of Congenital Platelet Disorders

Disease Platelet Count Platelet Major Clinical Features Inheri- Genetics Size tance Bernard-Soulier syndrome Low, Variable Very Homozygous – severe AR GPIbα (17p13), large thrombocytopenia, gi- GP Ibβ (22q11), ant platelets, defective or GPIX (3q21) ristocetin-induced mutations platelet aggregation. Heterozygous – Mild macrothrombocy- topenia, normal ristocetin-induced platelet aggregation Congenital amegakaryocytic Severe thrombocy- Normal Thrombocytopenia at AR c-mpl (1p34) thrombocytopenia topenia birth with progression gene mutations to other cytopenias

δγ Storage pool disorder Low, variable Normal Combined alpha and AD Unknown ? dense granule defi- ciency Dense granule deficiency (Idio- Normal Normal Normal to impaired AR, AD Unknown pathic δ-storage pool disease) ? platelet aggregation, dense granules and/or contents deficient Dense granule deficiency with Normal Normal Associated with AR HPS gene muta- pigment abnormalities ? Hermansky-Pudlak tions [(HPS1, (HPS) Syndrome or ADTB3A HPS3, Chediak Higashi Syn- HPS4) or CHS1 drome (CHS) which is gene] associated with giant granules in leukocytes

Contemporary Issues in Diagnostic Hemostasis and Thrombosis ASCP Annual Meeting, October 2011 Case #1 - Discussion 6

Familial thrombocytopenia with Low, variable Normal Defective aggregation, AD Transcription predisposition to acute dense granule defi- factor CBFA2 myelogenous (AML) ciency, predisposition (AML1)(21q22) to myelodysplasia and mutations AML

Glanzmann thrombasthenia Normal Normal Absent or severely de- AR αIIb or β3 muta- creased aggregation tions with all agonists except ristocetin due to dys- functional or deficient GPIIbIIIa Low, variable Large Gray platelets (severe AR, AR Unknown α-granule protein defi- ciency) and neutrophils (Romanovsky stains). Dense granule defi- ciency in dominant form Macrothrombocytopenia with Low, variable Large Morphologically abnor- AD? Unknown cytoskeletal abnormalities mal platelets with large membrane complexes, disarranged actin and tubulin, no shape change during aggrega- tion Montreal platelet syndrome Low, variable Giant Spontaneous platelet AD Unknown aggregation and de- creased platelet calpain activity MYH9 related diseases Low, variable Large Leukocyte inclusions ± AD MYH9 (22q12- nephritis, deafness, 13) gene muta- cataracts tions

In Suzanne’s case the findings of thrombocytopenia with large platelets and Dohle-like inclusions in the neutrophils suggests one of the nonmuscle myosin heavy chain IIa syndromes, which include May- Hegglin anomaly, Fechtner syndrome, Sebastian syndrome, Epstein syndrome. In 2003, these diseases were reported to represent variable expressions of a single disease due to mutations in the MYH9 gene encoding for the non-muscle myosin heavy chain IIA (NMMHC-IIA).7 The terms “MYH9 gene-related autosomal thrombocytopenia” and “MYH9-related macrothrombocytopenia” is now used for this disease. Suzanne does not have a history of renal disease or deafness, but may be too young to show the devel- opment of cararacts. Pending further studies of renal function and hearing, the May-Hegglin anomaly ap- pears most likely (Table II).

The May-Hegglin anomaly was initially described by German physician Richard May (1909) in a family with large platelets. In 1945, Swiss physician Robert Hegglin, identified Dohle-like inclusions in a family with dominatly-inherited macrothrombocytopenia, and the triad of thrombocytopenia, giant platelets, and Dohle-like inclusions subsequently became known as the May-Hegglin anomaly (MHA).8 The Epstein platelet syndrome was subsequently identified in 1972, the Fechtner syndrome in 1985, and the Sebas- tian platelet syndrome (SPS) in 1990. In 1990 the genetic abnormality in patients with all of these disease was linked to a region on the short arm of chromosome 22, later identified as the MYH9 gene.

Contemporary Issues in Diagnostic Hemostasis and Thrombosis ASCP Annual Meeting, October 2011 Case #1 - Discussion 7

The myosin superfamily comprises are a large class of ATP-dependent mechanoenzymes involved in muscle contraction and a wide variety of other essential cell motility processes. Of the 18 myosin classes, class II muscle myosins are the most numerous and best understood, since they are a major component of skeletal, cardiac, and smooth muscle. However, structurally similar myosin II molecules termed non- muscle myosin II (NMII) are also present in muscle, as well as all other eukaryotic cells. NMII is essential for all activities involving cell movement and/or changes in cell shape, including cell-cell and cell-matrix adhesion, cell migration, mitosis, differentiation, phagocytosis, cytokinesis, etc. NMII consists of two heavy and two light chains molecules that structurally resemble a “two-headed tadpole.”9 The heavy chain region of the complete molecule consists of two globular heads (heavy chain motor domains) con- taining actin-binding regions and ATPase enzyme domains. The motor domains are linked by short neck regions to a long alpha-helical coiled-coil rod tail domain, which ends in a short non-helical region. Two smaller light chain molecules wrap around the neck region and have regulatory functions. There are three different myosin heavy chain isomers (NMHC II-A, NMHC II-B, and NMHC II-C) that occur in different proportions in tissues. In muscle tissue, the head domains bind to actin molecules, while the antiparallel coiled coil tail domains of the tail regions interact to form a bipolar complex. Muscle contraction occurs through the ATPase enzyme actvity of the head domain, which initiates a conformational change in the molecule, sliding the actin molecules, and their attached actin strands, in an anti-parallel direction.9

More than 40 MYH9 mutations have been identified, but most patients with clinical manifestations have mutations in the motor domain of the head region, particularly point mutations at Arg902. The patho- physiology of the clinical manifestations resulting from MYH9 mutations is far from being understood, but it is thought that NMII may have a regulatory role in megakaryocyte development, especially inhibiting the formation of the proplatelet or the migration of megakaryocytes to the vascular space.10-14 The mutation may permit the premature release of large functionally compromised platelets with defective ability to un- dergo shape change. Glomerular podocyte formation is compromised in patients with the renal manifes- tations of an MYH9 mutation, presumably leading to nephritis. The mechanism of the cataract formation is unknown.15

MYH9-related diseases are classified as rare (incidence ~ 1:50,000), but it is believed that the disease may have a much higher incidence, with most cases misdiagnosed as chronic immune thrombocytopenia or another disease.2, 3, 16 In part, this is because the severity and phenotypic features of these diseases are extremely variable, even within members of the same family. The platelet count is usually between 30 x 109/L and 100,000 x 109/L and giant platelets are always present. Severe or fatal bleeding episodes are rare, but easy bruisability, epistaxis, and other manifestations of mild/moderate platelet function diseases is usually found. Menorrhagia and accompanying iron deficiency are frequent in women with the disease, and female patients may present with these findings. The severity of the renal impairment and the age of the onset of the cataracts and high-tone hearing loss is variable, with possible delayed presentation of the latter two symptoms until the 50s.

3. What additional laboratory studies should be obtained?

MYH9 mutation analysis has recently became commercially available and is indicated in the evaluation of patients with suspected MYH9-related disorders.17-19 The diagnostic evaluation of these patients should also include an audiogram, renal function assessment (BUN, creatinine and creatinine clearance, and urinalysis), and ophthalmologic screening for cataracts.

4. What other forms of therapy should be attempted?

Patients with inherited thrombocytopenia have normal platelet survival and will respond to platelet trans- fusion for major surgery, trauma, or bleeding episodes.3, 11 In these circumstances, a platelet count >100 x 109/L or greater is attempted. Patients with inherited thrombocytopenia do not have immune thrombocy- topenia, and will not respond to standard ITP therapies. However, Suzanne should be advised to avoid aspirin, aspirin-containing products, and other prescription drugs, “over-the-counter” drugs, and herbs that interfere with platelet function. Wearing a MedicAlert bracelet is also recommended.

Contemporary Issues in Diagnostic Hemostasis and Thrombosis ASCP Annual Meeting, October 2011 Case #1 - References 8

Final Diagnosis: Inherited macrothrombocytopenia, MYH9-related (May-Hegglin anomaly).

References:

1. Zandecki M, Genevieve F, Gerard J, Godon A. Spurious counts and spurious results on haematology analysers: a review. Part I: platelets. International journal of laboratory hematology. Feb 2007;29(1):4-20.

2. Israels SJ, Kahr WH, Blanchette VS, Luban NL, Rivard GE, Rand ML. Platelet disorders in children: A diagnostic approach. Pediatr Blood Cancer. Jun 2011;56(6):975-983.

3. Cox K, Price V, Kahr WH. Inherited platelet disorders: a clinical approach to diagnosis and management. Expert review of hematology. Aug 2011;4(4):455-472.

4. Streif W, Knofler R, Eberl W. Inherited disorders of platelet function in pediatric clinical practice: a diagnostic challenge. Klin Padiatr. May 2010;222(3):203-208.

5. Salles, II, Feys HB, Iserbyt BF, De Meyer SF, Vanhoorelbeke K, Deckmyn H. Inherited traits affecting platelet function. Blood Rev. May 2008;22(3):155-172.

6. Nurden P, Nurden AT. Congenital disorders associated with platelet dysfunctions. Thrombosis and haemostasis. Feb 2008;99(2):253-263.

7. Seri M, Pecci A, Di Bari F, et al. MYH9-related disease: May-Hegglin anomaly, Sebastian syndrome, Fechtner syndrome, and Epstein syndrome are not distinct entities but represent a variable expression of a single illness. Medicine (Baltimore). May 2003;82(3):203-215.

8. Saito H, Kunishima S. Historical hematology: May-Hegglin anomaly. American journal of hematology. Apr 2008;83(4):304-306.

9. De La Cruz EM, Ostap EM. Relating biochemistry and function in the myosin superfamily. Current opinion in cell biology. Feb 2004;16(1):61-67.

10. Pecci A, Bozzi V, Panza E, et al. Mutations responsible for MYH9-related thrombocytopenia impair SDF-1-driven migration of megakaryoblastic cells. Thrombosis and haemostasis. Aug 11 2011;106(4).

11. Balduini CL, Pecci A, Savoia A. Recent advances in the understanding and management of MYH9-related inherited thrombocytopenias. British journal of haematology. Jul 2011;154(2):161-174.

12. Kunishima S, Saito H. Advances in the understanding of MYH9 disorders. Curr Opin Hematol. Sep 2010;17(5):405-410.

13. Geddis AE. The regulation of proplatelet production. Haematologica. Jun 2009;94(6):756-759.

14. Chen Z, Shivdasani RA. Regulation of platelet biogenesis: insights from the May-Hegglin anomaly and other MYH9- related disorders. J Thromb Haemost. Jul 2009;7 Suppl 1:272-276.

15. Han KH, Lee H, Kang HG, et al. Renal manifestations of patients with MYH9-related disorders. Pediatr Nephrol. Apr 2011;26(4):549-555.

16. Althaus K, Najm J, Greinacher A. MYH9 related platelet disorders - often unknown and misdiagnosed. Klinische Padiatrie. May 2011;223(3):120-125.

17. Kopp JB, Winkler CA, Nelson GW. MYH9 genetic variants associated with glomerular disease: what is the role for genetic testing? Semin Nephrol. Jul 2010;30(4):409-417.

18. Knofler R, Streif W. Strategies in Clinical and Laboratory Diagnosis of Inherited Platelet Function Disorders in Children. Transfus Med Hemother. 2010;37(5):231-235.

19. James P, Di Paola J. The application of genetics to inherited bleeding disorders. . Jul 2010;16 Suppl 5:35-39.

Contemporary Issues in Diagnostic Hemostasis and Thrombosis ASCP Annual Meeting, October 2011 The Case of the Middle-Aged Ornithologist with Thrombocytopenia and Expressive Aphasia

Clinical History: Vivian Mouette paddled the canoe si- lently through the Dark Cypress Swamp in the early dawn light, high-powered binoculars at the ready. The 55 year-old leader of the Birdwatchers' Field Club of Mason City was in search of an elusive prey. She had caught a glimpse of an ivory-billed woodpecker, long thought to be extinct, while on a visit to Bayou DeView in Arkansas two years ago. Since then, she was obsessed with trying to find one in the swamps near Mason City, MO. Quietly rounding a bend in the stream, Vivian couldn’t believe what she was seeing. Sitting on the limb of a dead cy- press tree was a large black and white woodpecker with a bright red crest and a long, pointed, ivory-colored bill. She quickly snapped several pictures of the creature with the binocular’s built in 15MP camera before it flew away. As she took her cell phone from her backpack to inform her friend Gladys of the amazing discovery, she noticed that the fingers of her right hand were numb. Gladys answered her call after several rings, but Vivian was unable to speak and couldn’t move her tongue. Her daughter later found her in her home, sitting in a kitchen chair and moaning. She was still unable to speak but was apparently able to understand language. On arrival at the ED of a local hospital hospital, Vivian had a plate- let count of 13 x 109/L and was given two units of plate- lets. She was transferred to the Midwest Hemostasis Center with the diagnosis of acute onset aphasia and right-sided hemiparesis with thrombocytopenia. Case #2

Contemporary Issues in Diagnostic Hemostasis and Thrombosis ASCP Annual Meeting, October 2011 Case #2 - Clinical History 10

Vivian was fully functional prior to this event, and ischemic infarct in the left lentiform nucleus had no prior history of strokes or TIA’s. However, her and left basal ganglia with no evidence of hemor- past medical history did include Berger's disease, rhage. hypothyroidism, COPD, hyperlipidemia, rheumatoid arthritis. She was taking a number of drugs for the Laboratory Data: Essential laboratory results were treatment of these problems, including prednisone, as follows: WBC – 7.3 x109/L (3.6-9x109/L), Hgb – Simvastatin, Darvocet, Plaquenil, Pentoxyfillin, Cy- 11.0 g/dL (N 12.9 – 15.5g/dL), Hct – 34% (N 39– clobenzaprine, Zocor, Levothyroxine, Trental, Flex- 47%), RBC – 3.36x106/L (N 4.18 – 5.22x106/L), eril, Tricor, Ticlodipine, folic acid, methotrexate, hy- drochlorothiazide, and an Atrovent inhaler. She was MCV – 90.5 fL (N 84.0–100 fL), RDW – 15.8% (N 9 9 not allergic to any medications and did not use alco- 12–16%), platelets – 23x10 /L (N 140–440x10 /L), hol or illicit drugs. She had smoked two to three PT – 9.7 sec (N 13.5–16.5 sec), aPTT – 34.0 sec packs of cigarettes per day for greater than 20 (N 32 – 48 sec), fibrinogen – 380 mg/dL (N 1500 – years. Vivian had a family history of myocardial in- 3500 mg/dL), LDH – 481IU/L (N 105 – 250 IU), to- farctions and cerebrovascular accidents on the ma- tal bilirubin – 0.7 mg/dL (0.2 – 1.3 mg/dL), conju- ternal side of her family. gated bilirubin – 0 mg/dL (N 0.0 – 0.3 mg/dL), re- ticulocyte count – 4.1% (N 1-2%), D-dimer 1.14 Physical Examination: On admission to the Mid- west Hemostasis Center, Vivian had the following mcg/mL (normal < 0.41), haptoglobin < 20 mg/dL vital signs: blood pressure - 146/76 mmHg, heart (normal 50-150). BUN 15, and creatinine 1.09 with rate 67/min, temperature - 97.2oF, and respirations glucose of 90. AST 20, ALT 17, alkaline phos- -16/min. She was alert, awake, and oriented times phatase 117. Urinalysis revealed small leukocytes four but aphasic with receptive capabilities intact. with large blood, trace ketones, 1 + bacteria, 0-5 Her pupils were equally reactive to light and ac- WBC's and 50-100 RBCs. EKG with normal sinus commodation, and extraocular movements were in- rhythm, normal axis, isolated Q wave in leads III and tact. However, her tongue had a decreased range of IV, R wave progression. motion and was deviated to right. Decreased range of motion of tongue. A cardiovascular examination showed a regular rate and rhythm with no murmur, Learning Objectives: (1) Understand the etiology gallop or rub. There was no carotid bruit. A respira- of thrombocytopenia, (2) Understand how to exam- tory examination revealed expiratory wheezing over ine the peripheral blood smear, and the importance the posterior field and decreased breath sounds of peripheral blood smear examination in patients over anterior chest. The abdomen was soft, nondis- with thrombocytopenia, (3) Understand the patho- tended, and nontender. There was no hepatosple- genesis and clinical presentation of the thrombotic nomegaly to palpation. Ecchymosis noted over left microangiopathies, (4) Understand the biosynthesis lower quadrant and there were petechia over ante- and biological functions of von Willebrand factor, (5) rior chest and lower extremities bilaterally and right Understand the technical aspects, limitations, and and left shoulder bruising. A neurologic examination clinical interpretation of laboratory assays for throm- showed a possible mandibular or cranial nerve V botic microangiopathies. sensory loss with decreased mandible sensation on the left and right and a possible cranial nerve XII lesion as the tongue protruded, deviated to right, and had a decreased range of motion. Vivian also had expressive aphasia with markedly decreased right upper extremity and shoulder strength and 1/5 strength in the right lower extremity, 4/5 in left upper extremity, and 5/5 in left lower extremity. Touch sen- sation was uniformly decreased on the right. Re- flexes were 3+ in right upper and lower extremities. There was a negative Babinski and Vivian was able to sit up with assistance. A head CT revealed an

Contemporary Issues in Diagnostic Hemostasis and Thrombosis ASCP Annual Meeting, October 2011 Case #2 - Discussion 11

Questions:

1. What is the most likely diagnosis for Vivian?

The findings of severe thrombocytopenia associated with anemia and neurologic abnormalities suggests that Vivian has thrombotic thrombocytopenic (TTP, Moschcowitz syndrome).

TTP is a thrombotic disease first reported by Moschcowitz in 1924 in a 16 year-old girl with anemia, pete- chiae, and microscopic hematuria, was found to have disseminated microvascular hyaline platelet thrombi at autopsy.30 The hallmark of TTP is systemic intravascular platelet aggregation, leading to microvascular thrombosis, microangiopathic hemolytic anemia, thrombocytopenia, and organ dysfunc- tion. A classic pentad of microangiopathic hemolytic anemia, thrombocytopenia, CNS involvement, renal involvement and fever is found in some patients but the triad of microangiopathic hemolytic anemia, thrombocytopenia, and neurologic symptomatology is much more common.9, 15, 28, 40 The differentiation of TTP and HUS from other diseases, including infection, antiphospholipid syndrome, malignant hyperten- sion, and pregnancy-associated DIC syndromes, and other diseases can be difficult, and an atypical clinical presentation is not uncommon.2, 10, 22, 34, 37, 38

2. What is the most likely pathogenesis of Vivian’s illness?

Vivian was receiving the antiplatelet drug ticlodipine, which probably triggered the development of TTP.

The pathogenesis of TTP was unknown prior to the 1980‘s. The finding of large von Willebrand factor (vWF) multimers in the plasma, first reported in 1982 by Moake, was subsequently followed in 1998 by the discovery of a deficiency or functional abnormality of the enzyme von Willebrand factor (VWF)- cleaving metalloprotease (VWFCP, a disintegrin and metalloproteinase with a thrombospondin type 1 mo- tif, member 13, ADAMTS-13).6, 27, 29, 42

vWF is synthesized in endothelial cells and megakaryocytes as a large precursor molecule, pre-pro-vWF.25, 26, 43 Proteolytic cleavage of the signal peptide results in 280 kDa pro-vWF monomers that link by disulfide bonds to form large multimers and homodimers with molecular masses into millions of daltons. Ultralarge, proto-vWF (UL-vWF) molecules, formed when the pro peptide is cleaved, are stored in the Weibel-Palade bodies of endothelial cells and the alpha granules of platelets.25 UL-vWF is secreted into the plasma and mostly circulates in this form. However, encountering conditions of shear force, the molecule undergoes an unfolding conformational rearrangement, and binds to receptors on the endothe- lial cell, which expose the previously inaccessible Tyr1605-Met1606 peptide bond and permits cleavage by ADAMTS-13 into smaller subunits of two to 20 molecules. The smaller cleaved vWF molecules binds platelets less effectively. ADAMTS-13 is coded by a gene on chromosome 9 and is a member of the zinc- containing ADAMTS metalloproteinase family that share common structural features.

Most cases of TTP are idiopathic, but rare cases of congenital TTP have been described.(7) In addition, acquired, secondary TTP has been described in patients with many diseases and receiving some drugs. An increased incidence of TTP has been described in patients with pregnancy, cancer (especially adeno- carcinoma of the breast, GI tract, and prostate), hematopoietic stem cell transplantation, autoimmune disease, , and infection (HIV, Streptococcus pneumonia, ).40, 45 Drugs associ- ated with TTP include the thienopyridines (ticlopidine, clopidogrel), quinine, immunosuppressive drugs (mitomycin-C, cyclosporine, tacrolimus, deoxycoformycin, a-interferon, gemcitabine) and chemotherapeu- tic drugs (mitomycin-C, gemcitabine, cisplatin, tamoxifen, bleomycin, cytosine arabinoside, and daunomycin).4, 24 Idiopathic TTP is of autoimmune origin and associated with specific anti-ADAMTS-13 antibodies that inhibit ADAMTS-13 activity, leading to increased UL-vWF plasma levels and the formation of microthrombi, with subsequent microangiopathic hemolytic anemia and thrombocytopenia.8, 18, 36, 41 The hereditary form of TTP (Schulman-Upshaw syndrome), and most cases of chronic relapsing TTP, are

Contemporary Issues in Diagnostic Hemostasis and Thrombosis ASCP Annual Meeting, October 2011 Case #2 - Discussion 12

due to ADAMTS-13 gene mutations that lead to decreased enzyme activity.19, 21 The etiology of the sec- ondary, disease or drug-associated cases of TTP are more controversial, and associated with both microvascular endothelial cell damage and inhibitory anti-ADAMTS-13 antibodies.44

Ticlopidine-associated TTP was first reported in a European dialysis center in 1991, and in the United States in 1998.32, 44 Subsequent studies revealed an incidence of 1 in 1600 to 1 in 5000. It usually pre- sents 2 to 12 weeks after the initiation of drug therapy, and is accompanied by severe thrombocytopenia, marked serum LDH elevations, mild renal insufficiency, and ADAMTS-13 inhibitors. Ticlopidine- associated TTP has a good response to plasma exchange but a high incidence of relapse after reexpo- sure to a thienopyridine drug. Clopidogrel-associated TTP was reported in the United States soon after clopidogrel was FDA-approved in 1998, and reportedly has a much lower incidence, of about 1 in 12 mil- lion, approximately three times the overall incidence of TTP in the population.1, 44 The renal insufficiency in clopidogrel-associated TTP is usually severe, but the thrombocytopenia is mild, and the etiology is microvascular endothelial cell damage, rather than anti-ADAMTS-13 antibodies. Clopidogrel-associated TTP responds slowly to plasma exchange, are relapses are infrequent. The comparative features of TTP induced by clopidogrel and thienopyridine are shown in Table III.

Table III Ticlopidine- versus Clopidogrel-Associated TTP

Feature Ticlopidine-Associated TTP Clopidogrel-Associated TTP

Pathophysiology Anti-ADAMTS-13 antibody Microvascular endothelial damage Microvascular endothelial damage

ADAMTS-13 deficiency Present Absent

ADAMTS-13 inhibitors Present Absent

Time of onset after drug exposure 2 - 12 weeks Within 2 weeks

Renal Disease Absent to mild Severe

Thrombocytopenia Severe Mild

Survival S/P plasma exchange >90%, rapid response (days) 70%, very slow response (weeks)

Spontaneous relapse Occasional Infrequent

Relapse with exposure to the other High Low thienopyridine

3. What additional laboratory tests should be obtained to confirm the diagnosis?

Laboratory evaluation of patients with TTP usually reveals severe thrombocytopenia (20-50 x 109/L) with moderate anemia, and a variable white blood count.5, 33 Peripheral smear examination must be performed to confirm the diagnosis, evaluate red blood cell morphology, and rule out leukemia and other diseases. Fragmented red cells are usually frequent, but they may not be abundant during early disease stages. Unfortunately, there are no definitive guidelines regarding the number of schistocytes to exclude TTP from other diseases. The hemolytic anemia also results in elevations of serum lactic dehydrogenase (LDH), decreased haptoglobin levels, and moderate hyperbilirubinemia (2.5-4 mg/dL). A Coomb’s test should be obtained to exclude an autoimmune hemolytic anemia. D-dimers and fibrinogen degradation

Contemporary Issues in Diagnostic Hemostasis and Thrombosis ASCP Annual Meeting, October 2011 Case #2 - Discussion 13

products (FDPs) are elevated from the formation and breakdown of thrombi. The prothrombin time and aPTT are usually within normal limits, while fibrinogen is increased. The BUN and creatinine are elevated proportional to the severity of the renal injury. SLE and other autoimmune disease should be excluded, and serologic studies for HIV should be obtained. Bone marrow evaluation to exclude other causes of thrombocytopenia may be indicated, depending on the clinical picture and other laboratory findings. An image of Vivian’s peripheral blood is shown in Fig. 1.

Fig. 1. Peripheral blood smear, Wright-Giemsa stain, 1000x.

The modern era in TTP began with the discovery of the association with ADAMTS-13 and the develop- ment of assays for ADAMTS-13 activity and anti-ADAMTS-13 autoantibodies.14, 35, 39 Although the diag- nosis, treatment, and monitoring of TTP is still based on clinical symptomatology supplemented by the basic laboratory tests discussed above, ADAMTS-13-related assays are widely available through refer- ence laboratories, and are increasingly being performed by individual hospital laboratories. These assays are categorized in Table II, and include sequencing of the ADAMTS-13 gene, assays for ADAMTS-13 an- tigen and enzyme activity, and assays for the detection of anti-ADAMTS-13 neutralizing and non- neutralizing antibodies.35

ADAMTS-13 Antigen Assays Immunoassays for plasma ADAMTS-13 use monoclonal or polyclonal antibodies specific for normal or mutated portions of the the molecule. These assays are not clinically beneficial at the present time.

Contemporary Issues in Diagnostic Hemostasis and Thrombosis ASCP Annual Meeting, October 2011 Case #2 - Discussion 14

ADAMTS-13 Activity Current methods for the detection of ADAMTS-13 activity depend on the in vitro cleavage of a vWF sub- strate (purified, plasma-derived or recombinant multimeric vWF or synthetic vWF peptides that incorpo- rate the ADAMTS-13 cleavage site) by ADAMTS-13 activity in patient plasma.11, 16, 17, 20 The resulting vWF cleavage products are detected and quantified by direct or indirect methods, and the ADAMTS-13 activity is determined. The direct assays include electrophoresis (SDS agarose or SDS-PAGE) with Western blotting, and Fluorescence Resonance Energy Transfer [FRET], while indirect assay detection methods use collagen binding, ristocetin-inducted aggregation, or ELISA. Of these assays, the FRET as- says have achieved the most interest and are presently commercially available in kit form through three different manufacturers.

Förster resonance energy transfer, resonance energy transfer, electronic energy transfer) is an interac- tion between two colored molecules (chromophores) based on their proximity. This phenomen was origi- nally described by the German scientist Theodor Fӧrster in 1946, and is now a fundamental technique to study molecular interactions in the biosciences. The principle of Förster resonance energy transfer is that a donor chromopore in close proximity to an acceptor chromophore (< 10nm), transfers energy by a non- radiative mechanism termed dipole-dipole coupling. Since fluorescence emissions can be easily and ac- curately detected, the donor and acceptor chromophores are usually fluorescent molecules, and the term fluorescence resonance energy transfer (FRETS) is used. Since the emission spectrum of the acceptor is predominant when the chromopore molecules are in close proximity, but donor excitation occurs when they separated, an event such as molecular movement or a chemical reaction can be detected. The FRETS assays for ADAMTS-13 activity, as originally described by Kokame and co-workers utilizes a modified A2 domain vWF fragment of vWF in which two amino acid residues (Q1599 and N1610) that span the ADAMTS-13 cleavage site are replaced by florescent molecules [A2pr(Nma) and A2pr(Dnp]. When the vWF fragment is intact the Nma and Dnp groups are in close proximity, so that when Nma is excited at 340 nm, energy is transferred to the quencher, Dnp. However, if the molecules are separated by substrate cleavage, energy transfer does not occur, and Nma emits detectable fluorescent light at 440 mn.

Anti-ADAMTS-13 Antibodies Anti-ADAMTS-13 autoantibodies are the usual cause of acquired TTP and may be of the neutralizing or non-neutralizing type. Neutralizing antibodies directly bind to the ADAMTS-13 molecule and directly in- hibit enzyme activity. They can be detected by ELISA assays utilizing immobilized recombinant ADAMTS- 13, or by patient plasma titration with normal plasma, followed by measurements of ADAMTS-13 activity. Non-neutralizing antibodies accelerate the clearance of ADAMTS-13 or its binding to the endothelial cell surface without directly inhibiting enzyme activity. Non-neutralizing antibodies are detected by ELISA.

Genetic Analysis of the ADAMTS-13 Gene PCR amplification and conventional sequence analysis is used to characterize the ADAMTS-13 gene on chromosome 9q34 in patients with suspected congenital TTP. At this time, approximately 80 mutations and 20 non-synonymous single nucleotide polymorphisms (SNPs) have been described. The mutations are predominately missense mutations, but have also included nonsense mutations, deletions, insertions, and splice site mutations.19

4. What is the best treatment plan for Vivian?

The mortality of untreated TTP from multiple organ damage is as high as 90%, so aggressive therapy is usually undertaken to avoid death and ischemic consequences such as stroke, myocardial infarction, transient ischemic attacks (TIAs), bleeding, and other problems. Daily high-volume plasma exchange for about a week with fresh frozen plasma is the treatment of choice, usually with corticosteroid therapy.5, 23, 31 More recently, anti-CD20 monoclonal antibody (Rituximab) has been successfully used in patients un-

Contemporary Issues in Diagnostic Hemostasis and Thrombosis ASCP Annual Meeting, October 2011 Case #2 - Discussion 15

responsive to conventional treatment.3, 12, 13 The current mortality rate is reduced to about 10% with ap- propriate treatment, although disease relapse is seen in about 15% of patients.

Vivian underwent placement of a left external jugular vein catheter followed by daily plasmapheresis with FFP replacement. The platelet count gradually increased, and reached 150,000 x 109/L by day 10. Plas- mapheresis was then discontinued. Anticoagulation was started for DVT prophylaxis with 40 units Love- nox subcutaneously every day. However, an ultrasound of the head revealed a large clot around the left IJ internal jugular where the catheter was placed. After surgery to remove the line, a full dose of Lovenox (110 units subcutaneously daily) was began. Vivian slowly recovered was discharged to rehabilitation at a physical and occupational therapy facility. She continued to do well and returned home after approxi- mately 8 weeks. However, one week after returning home Vivian developed easy bruising and returned to the emergency room after passing a melanotic stool. She has found to have thrombocytopenia, with a relapse of her TTP. In spite of specific medication instructions on discharge, Vivian began taking all of her “usual” medications that were in her medicine cabinet, including Ticlodipine. The patient's daughter dis- covered the medication and disposed of it before the patient was discharged home for the second time.

Final Diagnosis: Thrombotic (TTP), Ticlopidine-associated

References

1. Bennett CL et al. Thrombotic thrombocytopenic purpura associated with clopidogrel. N Engl J Med 342: 1773-1777, 2000.

2. Bindi ML, Mazzoni A, Bisa M, Grazzini T, Esposito M, Meacci L, Mozzo R, Scatena F, and Biancofiore G. The chal- lenges of diagnosing thrombotic thrombocytopenic purpura in the critically ill. A case report. Transfus Apher Sci 43: 167-170, 2010.

3. Bresin E et al. Rituximab as pre-emptive treatment in patients with thrombotic thrombocytopenic purpura and evi- dence of anti-ADAMTS13 autoantibodies. Thromb Haemost 101: 233-238, 2009.

4. Dlott JS, Danielson CF, Blue-Hnidy DE, and McCarthy LJ. Drug-induced thrombotic thrombocytopenic purpura/ hemolytic uremic syndrome: a concise review. Ther Apher Dial 8: 102-111, 2004.

5. Franchini M, Zaffanello M, and Veneri D. Advances in the pathogenesis, diagnosis and treatment of thrombotic thrombocytopenic purpura and hemolytic uremic syndrome. Thromb Res 118: 177-184, 2006.

6. Furlan M et al. von Willebrand factor-cleaving protease in thrombotic thrombocytopenic purpura and the hemolytic- uremic syndrome. N Engl J Med 339: 1578-1584, 1998.

7. Galbusera M, Noris M, and Remuzzi G. Inherited thrombotic thrombocytopenic purpura. Haematologica 94: 166-170, 2009.

8. George JN. The role of ADAMTS13 in the pathogenesis of thrombotic thrombocytopenic purpura-hemolytic uremic syndrome. Clin Adv Hematol Oncol 3: 627-632, 2005.

9. George JN. The thrombotic thrombocytopenic purpura and hemolytic uremic syndromes: overview of pathogenesis (Experience of The Oklahoma TTP-HUS Registry, 1989-2007). Kidney Int Suppl S8-S10, 2009.

10. Gerth J, Schleussner E, Kentouche K, Busch M, Seifert M, and Wolf G. Pregnancy-associated thrombotic thrombo- cytopenic purpura. Thromb Haemost 101: 248-251, 2009.

11. Groot E, Hulstein JJ, Rison CN, de Groot PG, and Fijnheer R. FRETS-VWF73: a rapid and predictive tool for thrombotic thrombocytopenic purpura. J Thromb Haemost 4: 698-699, 2006.

Contemporary Issues in Diagnostic Hemostasis and Thrombosis ASCP Annual Meeting, October 2011 Case #2 - References 16

12. Hagel S, Jantsch J, Budde U, Kalden JR, Eckardt KU, and Veelken R. Treatment of acquired thrombotic thrombo- cytopenic purpura (TTP) with plasma infusion plus rituximab. Thromb Haemost 100: 151-153, 2008.

13. Hull MJ, and Eichbaum QG. Efficacy of rituximab and concurrent plasma exchange in the treatment of thrombotic thrombocytopenic purpura. Clin Adv Hematol Oncol 4: 210-214; discussion 217-218, 2006.

14. Just S. Methodologies and clinical utility of ADAMTS-13 activity testing. Semin Thromb Hemost 36: 82-90.

15. Kiss JE. Thrombotic thrombocytopenic purpura: recognition and management. Int J Hematol 91: 36-45, 2010.

16. Kokame K, Nobe Y, Kokubo Y, Okayama A, and Miyata T. FRETS-VWF73, a first fluorogenic substrate for AD- AMTS13 assay. Br J Haematol 129: 93-100, 2005.

17. Kremer Hovinga JA, Mottini M, and Lammle B. Measurement of ADAMTS-13 activity in plasma by the FRETS- VWF73 assay: comparison with other assay methods. J Thromb Haemost 4: 1146-1148, 2006.

18. Lian EC. Pathogenesis of thrombotic thrombocytopenic purpura: ADAMTS13 deficiency and beyond. Semin Thromb Hemost 31: 625-632, 2005.

19. Lotta LA, Garagiola I, Palla R, Cairo A, and Peyvandi F. ADAMTS13 mutations and polymorphisms in congenital thrombotic thrombocytopenic purpura. Hum Mutat 31: 11-19, 2010.

20. Mahdian R, Rayes J, Girma JP, Houllier A, Obert B, Meyer D, and Veyradier A. Comparison of FRETS-VWF73 to full-length VWF as a substrate for ADAMTS13 activity measurement in human plasma samples. Thromb Haemost 95: 1049-1051, 2006.

21. Manea M, and Karpman D. Molecular basis of ADAMTS13 dysfunction in thrombotic thrombocytopenic purpura. Pediatr Nephrol 24: 447-458, 2009.

22. Mar N, and Mendoza Ladd A. Acquired thrombotic thrombocytopenic purpura: puzzles, curiosities and conundrums. J Thromb Thrombolysis 31: 119-121, 2011.

23. Marn Pernat A et al. Membrane plasma exchange for the treatment of thrombotic thrombocytopenic purpura. Ther Apher Dial 13: 318-321, 2009.

24. Medina PJ, Sipols JM, and George JN. Drug-associated thrombotic thrombocytopenic purpura-hemolytic uremic syndrome. Curr Opin Hematol 8: 286-293, 2001.

25. Metcalf DJ, Nightingale TD, Zenner HL, Lui-Roberts WW, and Cutler DF. Formation and function of Weibel-Palade bodies. J Cell Sci 121: 19-27, 2008.

26. Michaux G, and Cutler DF. How to roll an endothelial cigar: the biogenesis of Weibel-Palade bodies. Traffic 5: 69- 78, 2004.

27. Moake JL. von Willebrand factor in the pathophysiology of thrombotic thrombocytopenic purpura. Clin Lab Sci 11: 362-364, 1998.

28. Moake JL. von Willebrand factor, ADAMTS-13, and thrombotic thrombocytopenic purpura. Semin Hematol 41: 4-14, 2004.

29. Moake JL et al. Unusually large plasma factor VIII:von Willebrand factor multimers in chronic relapsing thrombotic thrombocytopenic purpura. N Engl J Med 307: 1432-1435, 1982.

30. Moschcowitz E. An acute febrile pleiochromic anemia with hyalin thrombosis of the terminal arterioles and capillar- ies. Arch Intern Med 36: 89-93, 1925.

31. Murrin RJ, and Murray JA. Thrombotic thrombocytopenic purpura: aetiology, pathophysiology and treatment. Blood Rev 20: 51-60, 2006. Case #2 - References 17

32. Page Y, Tardy B, Zeni F, Comtet C, Terrana R, and Bertrand JC. Thrombotic thrombocytopenic purpura related to ticlopidine. Lancet 337: 774-776, 1991.

33. Park YA, Waldrum MR, and Marques MB. Platelet count and prothrombin time help distinguish thrombotic thrombo- cytopenic purpura-hemolytic uremic syndrome from disseminated intravascular in adults. Am J Clin Pathol 133: 460-465, 2010.

34. Patel A, and Patel H. Thrombotic thrombocytopenic purpura: the masquerader. South Med J 102: 504-509, 2009.

35. Peyvandi F, Palla R, Lotta LA, Mackie I, Scully MA, and Machin SJ. ADAMTS-13 assays in thrombotic thrombocy- topenic purpura. J Thromb Haemost 8: 631-640, 2010.

36. Sadler JE. Von Willebrand factor, ADAMTS13, and thrombotic thrombocytopenic purpura. Blood 112: 11-18, 2008.

37. Sarode R. Atypical presentations of thrombotic thrombocytopenic purpura: a review. J Clin Apher 24: 47-52, 2009.

38. Thachil J. The difficult distinction between antiphospholipid syndrome and thrombotic thrombocytopenic purpura. Br J Haematol 149: 294-295, 2010.

39. Tripodi A et al. Second international collaborative study evaluating performance characteristics of methods measur- ing the von Willebrand factor cleaving protease (ADAMTS-13). J Thromb Haemost 6: 1534-1541, 2008.

40. Tsai HM. Pathophysiology of thrombotic thrombocytopenic purpura. Int J Hematol 91: 1-19, 2010.

41. Tsai HM. Thrombotic thrombocytopenic purpura: a thrombotic disorder caused by ADAMTS13 deficiency. Hematol Oncol Clin North Am 21: 609-632, v, 2007.

42. Tsai HM, and Lian EC. Antibodies to von Willebrand factor-cleaving protease in acute thrombotic thrombocytopenic purpura. N Engl J Med 339: 1585-1594, 1998.

43. Wang JW, and Eikenboom J. and Weibel-Palade bodies. Hamostaseologie 30: 150-155, 2010.

44. Zakarija A et al. Ticlopidine- and clopidogrel-associated thrombotic thrombocytopenic purpura (TTP): review of clinical, laboratory, epidemiological, and pharmacovigilance findings (1989-2008). Kidney Int Suppl S20-24, 2009.

45. Zhou Z, Nguyen TC, Guchhait P, and Dong JF. Von Willebrand factor, ADAMTS-13, and thrombotic thrombocy- topenic purpura. Semin Thromb Hemost 36: 71-81, 2010.

Contemporary Issues in Diagnostic Hemostasis and Thrombosis ASCP Annual Meeting, October 2011 The Case of the Italian Baker with Multiple Myocardial Infarctions and Stent Thrombosis

Clinical History: In Mason City, MO, many folks begin their day with a trip to Guido's Pasticceria and Caffe' for an espresso or cappuccino with some homemade pigno- lata, ciarduna, bomboloni con la crema, or a slice of Torta della Nonna. Founded in the 1950’s by Guido San- tamarin and his wife Maria, immigrants from Treviso, It- aly, the business is now ran by Guido’s son, Antonio and his wife Lisa. Antonio, now 47, practically grew up in the bakery, often sleeping on bags of flour while his father baked bread in their brick oven for early morning deliv- ery. Their hard work has resulted in an genuine old world Italian cafe, with Italian marble floors, stained glass win- dows and a stamped copper ceiling. Mason city citizens daily line up around the block to get into the pasticceria.

One Friday morning, Antonio arrived at the bakery even earlier than usual to begin preparing for the busy week- end pastry sales. He was filling a large tray of cannoli shells when he developed severe substernal chest pres- sure that radiated around his left side to his back and neck. The pain was associated with lightheadedness, tingling in his right hand, left arm pain, shortness of breath, and palpitations. He stopped filling the shells and took 2 nitroglycerine tablets with some relief. He imme- diately called the Mason City EMS and received aspirin, an additional nitroglycerin tab, and a lopressor en route to Mason City Medical Center. In the ED he was started on a heparin drip, and received nitropaste, 10mg of IV metoprolol, and 20U insulin for a blood glucose > 500 mg/dL prior to being transferred to the Cardiac ICU. Case #3 Case #3 - Clinical History 19

By the time cardiologist Dr. Renee Hartz was con- PO, q 8 hr), clonidine (0.3 mg, 1 tab, PO, q 8 hr), sulted, Antonio only had minimal residual back pain, insulin glargine (82 Units, SQ, bedtime), famotidine and no chest pain, shortness of breath, or nausea/ (20 mg, 1 tab, PO, bedtime), omega-3 polyunsatu- vomiting. However, upon reviewing Antonio’s re- rated fatty acids (2,000 mg, PO, bid), chlorthalidone cords, Dr. Hartz found an alarming past medical his- (25 mg, 1 tab, PO, bid), and pravastatin (40 mg, 1 tory, with hypertension, diabetes mellitus, hypercho- tab, PO, bedtime). lesterolemia, coronary artery disease, multiple (5) myocardial infarctions, peripheral vascular disease, Physical Examination: A physical examination re- congestive heart failure (EF of 40%), chronic kidney vealed: blood pressure - 150/80 mm Hg, heart rate - disease (stage III), moderate obesity, and nonsus- 80 bpm, and respirations - 16/min. Antonio’s lungs tained ventricular tachycardia (NSVT) status-post were clear to auscultation and percussion, and he placement of an automatic implanted cardiac defibril- had a regular cardiac rhythm with no murmurs or lator (AICD). He had previously underwent multiple regurgitation. balloon angioplasty and stenting procedures for his coronary artery disease, and had recently under- Laboratory Data: An ECG was non-ischemic but gone a percutaneous transluminal coronary angio- revealed left ventricular hypertrophy with strain. Tro- plasty with placement of a drug-eluting stent placed ponin I levels were elevated, at 1.0 ng/mL. Dr. Hartz for instent thrombosis. In addition to multivitamins, makes a diagnosis of a non-ST elevation myocardial Antonio’s medications include aspirin (325 mg, 1 tab, infarction (non-STEMI MI) but is concerned about PO, daily), metoprolol (100 mg, 1 tab, PO, every 12 recurrent stent restenosis or plaque rupture leading hours), nitroglycerin (0.4 mg, 1 tab, SL, every 5 min- to anginal symptoms. She knows that Antonio is very utes, PRN: for chest pain), clopidogrel (75 mg, 1 tab, compliant about taking his medications, and also PO, daily), isosorbide dinitrate (30 mg, 1 tab, PO, wonders about resistance to clopidogrel and aspirin. every 8 hours, 90 tab), spironolactone (25 mg, 1 tab, Finding no record of a VerifyNow assay in Antonio’s PO, bid, 60 tab), lisinopril (40 mg, 1 tab, PO, daily), records, she orders P2Y12 and aspirin resistance insulin aspart (100 u/ml subcutaneous solution, 6 assays from the coagulation laboratory. The results Units, SQ, ac tid & hs), hydralazine (50 mg, 1 tab, are as follows:

Table IV Antonio’s Accumetric VerifyNow Assay Results

P2Y12 Assay Procedure Result Normal Values

P2Y12 % Inhibitor 15% -

P2Y12 Assay 252 PRU -

Baseline 295 PRU 194-418 PRU

Aspirin Assay Procedure Result Normal Values

Aspirin Assay 422 ARU 620-672 ARU

Contemporary Issues in Diagnostic Hemostasis and Thrombosis ASCP Annual Meeting, October 2011 Case #3 - Discussion 20

Questions

2. Discuss the pathophysiology of arterial thrombosis and the management of atherothrombotic dis- ease, including the use of antiplatelet drugs.

Atherosclerosis (arteriosclerotic vascular disease, ASVD, hardening of the arteries) is the most common vas- cular disease in the world. It is a chronic disease affecting the large and medium-sized arteries,that requires decades to develop. The pathogenesis is extremely complex, but a chronic inflammatory response in the wall of the artery causes the accumulation of fatty substances, especially cholesterol, macrophages, calcium, and other substances, to form a firm structure along the intraluminal wall of the vessel termed a plaque. The for- mation of atherosclerotic plaques is accentuated by low-density lipoproteins and countered by high density lipoproteins (HDL). Stable plaques rich in smooth muscle narrow the lumen and decrease the elasticity of the artery, but generally do not produce clinical symptomatology. However, plaques rich in macrophages and foam cells (unstable plaque, vulnerable plaque) may have a weak, narrow layer of tissue (i.e., fibrous cap) separating them from the lumen. These unstable plaques can rupture, expose thrombogenic material to the circulation, initiate activation of the platelets and the coagulation system, and promote the development of a blood clot (). The sudden intraluminal occlusion of an artery can rapidly lead to cardiac ischemia/ necrosis, with the clinical consequences of a angina pectoris, myocardial infarction, stroke, or peripheral vas- cular disease, depending on the anatomic location of the lesion. The thrombi can also detach and move down the bloodstream to smaller arteries anywhere in the body (thromboembolic disease). The risk of devel- oping atherosclerosis is determined by genetic factors in combination with multiple acquired risk factors, in- cluding body weight, life style, diet, hypertension, tobacco smoking, other diseases (diabetes, dyslipopro- teinemia), stress, medications, etc. Diet and behavioral modification is initially used to prevent or alleviate atherosclerosis, but a combination of dietary modification and drugs (i.e., statins, niacin, intestinal cholesterol absorption-inhibiting substances, Omega-3 oils) are used for patients with more advanced disease to promote a more favorable lipoprotein ra- tio. Patients with severe acute or life-threatening disease may require angioplasty with stent placement to expand the narrowed arteries, or bypass surgery to open alternate sources of blood flow around the diseased vessel. Since anticoagulants are not effective in arterial disease, therapy with antiplatelet drugs are used as a prophylactic measure in patients at risk for thrombotic cerebrovascular or cardiovascular disease, or secon- darily in patients status-post treatment by medical and surgical means. The antiplatelet drugs currently in use are briefly reviewed below. 2. Describe the pharmacological effects, medical indications, and side effects of antiplatelet drugs, in- cluding aspirin, clopidogrel, ticlopidine, prasugrel, and dipyridamole.

Antiplatelet drugs interact with different pathways of platelet metabolism to prevent platelet activation.1-5 Common targets of current antiplatelet drugs include cyclooxygenase 1 (COX-1)(aspirin), P2Y12 receptor (ti- clopidine, clopidogrel, prasugrel), phosphodiesterase (Cilostazol), glycoprotein IIb/IIIA receptor (abciximab, eptifibatide, tirofiban), and the thromboxane receptor (Terutroban).3, 6, 7 Another antiplatelet drug (Dipyrida- mole) acts through multiple mechanisms to inhibit adenosine reuptake. The major properties of these drugs is summarized in Table V. These drugs comprise a multibillion dollar industry, and may new antiplatelet drugs are in development or undergoing clinical trials.8-10

Aspirin Salicylates are among the oldest and most versatile human drugs. Salicylates are heterocyclic compounds present in several common plants, including willow, birch, beech, and popular. Due to its potent analgesic, antipyretic, and antiinflammatory effects, salicyclic acid has been used as a human pharmacological agents for at least 2400 years. Acetylsalicylic acid, in which a carboxyl group is substituted for the hydroxy moiety of salicylic acid (o-hydroxybenzoic acid), was originally synthesized by French chemist Charles Frederic Ger- hardt in the mid-1850‘s and termed "salicylic-acetic anhydride." In 1897, chemists at the German firm Bayer AG resynthesized acetylsalicylic acid (2-acetoxybenzoic acid) from salicin and found that it had similar anal-

Contemporary Issues in Diagnostic Hemostasis and Thrombosis ASCP Annual Meeting, October 2011 Case #3 - Discussion 21

gesic properties but less gastric toxicity than salicyclic acid.11, 12 The compound was named aspirin “acetyl” and the old German name for salicylic acid “spirsäure,” The identity of the Bayer chemist responsible for the discovery is a matter of controversy. However, there is considerable evidence that Jewish chemist Arthur Ei- chengrün actually made the discovery, but that records of the discovery were expunged by the Nazi regime, and credit was given to another chemist Felix Hoffman. belived that a There is a controversy whether Felix Hoffmann, but the later claimed that he was the lead investigator and records of his contribution were ex- punged under the Nazi regime.13 Aspirin quickly become one of the worlds most widely used drugs, with nearly 40,000 tons consumed yearly.14

Table V FDA-Approved Antiplatelet Drugs

Class Drug Mechanism Route Side Effects

Cycloxxygenase Inhibi- Aspirin Irreversible Oral Bleeding tors acetylation of cyclooxyge- GI toxicity nase 1

ADP Receptor Inhibitors Clopidogrel (Plavix) Irreversible Oral Bleeding inhibition of P2Y12 receptor Neutropenia TTP

Ticagrelor Reversible, allosteric P2Y12 Oral Dyspnea (Brilinta) inhibitor Bleeding Rash

Prasugrel (Effient) Irreversible Oral Bleeding inhibition of P2Y12 receptor

Ticlopidine (Ticlid) Irreversible Oral Bleeding inhibition of P2Y12 receptor Rash Neutropenia TTP

GPIIb/IIIa Inhibitors Abciximab (ReoPro) Integrin αIIbβ3 IV Bleeding antagonist Thrombocytopenia Pseudothrombocytopenia

Eptifibatide (Integrilin) Integrin αIIbβ3 IV Bleeding antagonist Thrombocytopenia Pseudothrombocytopenia

Tirofiban (Aggrastat) Integrin αIIbβ3 IV Bleeding antagonist Thrombocytopenia Pseudothrombocytopenia

ADP Reuptake Inhibitors Dipyridamole (Persan- Inhibition of ADP uptake; Oral Headache tine) Inhibition of phosphodies- Dizziness terase Hypotension GI toxicity

Phosphodiesterase In- Cilostazol (Pletal) Inhibition of phosphodies- Oral Bleeding hibitors terase Headache Diarrhea Palpitations Dizziness Rash Pancytopenia

Contemporary Issues in Diagnostic Hemostasis and Thrombosis ASCP Annual Meeting, October 2011 Case #3 - Discussion 22

In the United States, at least 26 million individuals ingest aspirin daily to prevent heart disease and stroke. The antithrombotic effects of aspirin were not recognized until the mid-20th century, when Gibson, and later Craven reported its beneficial effect in patients with myocardial infarction, strokes, and other vascular diseases.15, 16

The pharmacologic effects of aspirin remained unexplained until 1971, when British pharmacologist John Vane reported that it interferes with the synthesis of prostaglandins and thromboxane.12, 17 Prostaglandins are fatty acid derivatives that, together with the thromboxanes and prostacyclins, comprise the prostanoids. These com- pounds exert a wide variety of important physiologic effects by acting as autocrine or paracrine signaling agents. At the cellular level, the enzyme phospholipase A generates arachidonic acid from membrane phoshop- lipids, which serves an intermediary for additional chemical reactions. Leukotrienes are generated from arachi- donic acid (AA) through the actions of the enzymes 12-lipoxygenase and 5-lipoxygenase, while prostaglandin H (PGH) synthase (cycloxygenase, COX, PHS) and other synthases. COX is an unusual enzyme with two active sites. A cycloxygenase site converts arachidonic acid into prostaglandin G2 (PGG2), while a heme site with per- oxidase activity converts the PGG2 into prostaglandin H2 (PGH2). PGH2 is further modified by other synthases into additional prostanoids, including PGD2, PGE2, PGF2α, PGI2 (prostacyclin), and thromboxane A2 (TXA2). Three isoenzymes of COX have been discovered. COX-1 is a constitutive enzyme expressed in nearly all mammalian cells, while COX-2 is inducible by inflammatory stimuli, and expressed by microvascular endothelial cells, macrophages, and other cells at the sites of injury or inflammation, where it generates prostaglandin in- flammatory mediators. COX-3 (COX-1b, COX-1v) is a splice variant of COX-1. Aspirin irreversibly acetylates serine 529 of COX-1 and serine 516 of COX-2.18, 19 The acetylated enzymes cannot convert arachiconic acid to PGH2, and additional prostanoids cannot be generated. The inhibition of COX-1 is responsible for the anti- thrombotic effects of aspirin, while the inhibition of COX-2 explains its antiinflammatory role. The major role of platelet COX-1 is the generation of thromboxane A2, a potent mediator of vasoconstriction and platelet aggre- gation. However, since COX-1 is not involved in other mechanisms of platelet activation through physical forces or other platelet membrane receptors, aspirin is a relatively weak inhibitor of platelet function.

Aspirin is rapidly absorbed in the stomach and upper gastrointestinal tract, appears in the circulation within 20 minutes of ingestion, and reaches a peak plasma level within 30-40 minutes. Aspirin-induced platelet inhibition largely occurs in the portal circulation, can be detected within one hour, and persists for the entire life of the platelet (8-10 days). However, in view of the continuing production and release of platelets, platelet COX func- tion recovers by approximately 10% per day after a single dose of aspirin, and the bleeding time normalizes in 3 to 4 days. The appropriate dose of aspirin has been the subject of extensive research studies. A relative low daily dose of aspirin (i.e., 81 mg) has been generally accepted as optimal for the general population, since it completely inhibits TXA2-mediated vasoconstriction and platelet aggregation while leaving the endothelial cell prostacyclin generation unaffected. However, in patients with acute vascular injury, higher-dose aspirin (i.e., 325 mg) may also prevent the deliterious effect of collagen-induced platelet aggregation. Daily ASA doses greater than 325 mg are associated with significant gastric toxicity.

Clopidogrel Clopidogrel bisulfate, marketed under the trade name Plavix by Bristol-Myers Squibb and Sanofi-Aventis, is an oral antiplatelet agent in the thienopyridine class of drugs. Since 2007, it has been the second most widely pre- scribed drug worldwide, reaching US sales of approximately $4. 2 billion in 2009. Clopidogrel was discovered by Jean-Pierre Maffrand, Director of Discovery Research at the pharmaceutical company Sanofi-Aventis. Plavix is a prescription medication available in tablets that supply the molar equivalent of 75 or 300 mg of clopidogrel. The medical applications of clopidogrel, alone or in combination with aspirin and other antithrombotic drugs, include: (1) the prevention of vascular ischemic events in patients with symptomatic atherosclerosis, (2) treat- ment of acute coronary syndrome without ST-segment elevation (NSTEMI), and (3) treatment of ST elevation MI (STEMI), It is also used for the prevention of thrombosis after the placement of intracoronary stents, and in patients with a history of aspirin-induced gastric ulcers who require antiplatelet drug therapy.

Clopidogrel is a prodrug that is metabolized by a hepatic CYP450 cytochrome isoenzymes (CYP3A4, CYP3A5, CYP2C19) to produce an active thiol metabolite that irreversibly inhibits ADP binding by forming a disulfide

Contemporary Issues in Diagnostic Hemostasis and Thrombosis ASCP Annual Meeting, October 2011 Case #3 - Discussion 23

bond between two groups in the extracellular domain of the platelet low affinity ADP receptor P2Y12.20 This inhibits ADP-mediated activation of the GPIIb/IIIa complex.21 Platelet function inhibition is present as soon as two hours after clopidogrel ingestion and reaches a steady state of 40% to 60% inhibition between Day 3 and Day 7. Approximately 5 days is required for platelet function to return to baseline following discontinuation of clopidogrel administration. Hemorrhage is the major adverse effect of clopidogrel, with an 2% incidence of gastrointestinal hemorrhage annually. Other serious, but rare adverse effects include neutropenia (0.05% in- cidence) and TTP (0.004% incidence). Clopidogrel also interacts with many other drugs, especially those me- tabolized by CYP450 enzymes. Approximately 2% to 14% of the population have CYP2C19 polymorphisms that prevent the conversion of clopidogrel base into its active metabolite.22 As of May, 2009, the FDA has re- quired a black box warning to the drug label recommending CYP2C19 genotyping to identify poor clopidogrel metabolizers (http://www.fda.gov/NewsEvents/Newsroom/PressAnnouncements/ucm204253.htm).

Ticagrelor Ticagrelor is a new platelet ADP inhibitor approved by the FDA on July 12, 2011, and marketed under the trade name of Brilinta by AstraZeneca (Wilmington, Del.) in the US. Ticagrelor is a reversible P2Y12 antago- nist that binds to a different site than clopidogrel.23 In contrast to clopidogrel, it also does not require hepatic activation. Ticagrelor is rapidly absorbed from the gastrointestinal tract, reaches peak plasma concentration in about 1.5 hours, and has a plasma half-life of approximately 7 hours. It is metabolized by hepatic CYP450 cytochrome enzymes and excreted mainly in the bile and feces. The major side effect is dyspnea, but as will all platelet agonists, there is a risk of bleeding. Reportedly, ticagrelor has a lower mortality rate than clopidogrel.24, 25 The use of Ticagrelor is contraindicated in patients with active bleeding or a history of intrac- ranial bleeds, reducted liver function, or the concomitant use of other drugs that interfere with CYP3A4.

Ticlopidine Ticlopidine (ticlid) is a thienopyridine ADP receptor inhibitor discovered by Dr. Jean-Pierre Maffrand that is similar to clopidogrel in structural and pharmacological properties. However, because of a relatively higher incidence of neutropenia, , TTP, and gastric hemorrhage, it is not widely used.

Prasugrel Prasugrel is a third member of the thienopyridine class of platelet ADP receptor inhibitors. It was developed by the Japanese pharmaceutical company Daiichi Sankyo Co. and is marketed in the United States by Eli Lilly and Company under the trade name Effient. It was approved by the FDA in July, 2009 for patients with thrombotic cardiovascular diseases planned for management by percutaneous coronary intervention (PCI). Prasugrel is a more potent and clinically effective platelet inhibitor than clopidogrel, but is also associated with a higher risk of adverse bleeding. However, prasugrel is metabolized by the CYP3A4 and CYP2B6 iso- forms of CYP450, and genetic polymorphism has not been demonstrated.

Dipyridamole Dipyridamole (Permole, Persantine) is a vasodilator first used in the early 1960s for patients with chest pain. It was reported to have antiplatelet activity in 1968. Dipyridamole exerts multiple effects that decrease plate- let function. It inhibits the uptake of adenosine into platelets and endothelial cells and inhibits several en- zymes, including thromboxane synthetase, adenosine deaminase, and cAMP/cGMP phosphodiesterases. These actions lead to decreased availability of TXA2 and increased local concentrations of adenosine which inhibit platelet aggregation to PAF, collagen, and ADP. Dipyridamole is highly bound to plasma proteins and metabolized in the liver. Dipyridamole is used in combination with warfarin for postoperative thromboembolic prophylaxis in patients receiving prosthetic heart valves. A combination of modified release dipyridamole (Ag- grenox) and aspirin is also used for the prevention of stroke and transient ischemic attacks.

Abciximab Abciximab (RePro, Eli Lily) consists of the Fab fragments of a monoclonal anti-glycoprotein IIb/IIIA antago- nist. The drug has a short plasma half-life but induces an inhibition of platelet function that persists for at least 48 hours. Abciximab is primarily used as an antithrombotic agent in patients undergoing percutaneous

Contemporary Issues in Diagnostic Hemostasis and Thrombosis ASCP Annual Meeting, October 2011 Case #3 - Discussion 24

transluminal coronary angioplasty (PTCA) with or without stent placement. Severe thrombocytopenia is a rare but serious complication with this agent.

Tirofiban Tirofiban (Aggrastat, Medicure Pharma) is a synthetic, rapid acting, non-peptide glycoprotein IIb/IIIA antago- nist with a short half-life modeled from an anticoagulant present in the venom of the saw-scaled viper (Echis carinatus). It is used in combination with heparin and aspirin in the management of patient with unstable an- gina or non-ST-wave myocardial infarction planed for percutaneous transluminal coronary angioplasty (PTCA).

Eptifibatide Eptifibatide (Integrilin, Millennium Pharmaceuticals) is a rapid acting glycoprotein IIb/IIIA antagonist used in combination with heparin and aspirin or clopidogrel in the management of patients with nstable angina or non-ST-segment-elevation (e.g., non-Q-wave) myocardial infarction. Structurally, it is an unusual cyclic hep- tapeptide modeled from a compound present in the venom of the southeastern pygmy rattlesnake (Sistrurus miliarius barbouri). Bleeding and hypotension are serious side effects of eptifibatide.

3. Define “aspirin resistance” and “clopidogrel resistance” and discuss the significance of these phenomenon in the care of patients with thrombotic disease.

Clinically, “resistance,” “nonresponsiveness”, or “failure” to aspirin and/or clopidogrel refers to the develop- ment of a thrombotic vascular event prescribed an adequate dose of the drug(s).26-29 This may occur due to poor patient compliance or suboptimal drug absorption or other pharmacokinetic factors.30 Genetic variability can also affect the magnitude of response in individual patients. In the laboratory, the time of specimen col- lection and the method of platelet function quantitation can cause variability in the results. Unfortunately, the reported incidence of these phenomenon has varied widely in different studies, depending on the patient population and the means of determination. Until precise criteria for these phenomenon are established, “nonresponsiveness” is preferred over other terminology.

Clinically, thrombosis occurs in approximately 13% of patients receiving aspirin therapy (Antithrombotic Trial- ist Collaboration. Collaborative meta-analysis of randomized trials of antiplatelet therapy for prevention of death, myocardial infarction, and stroke in high risk patients.31 However, attempts to quantify the effects of aspirin by laboratory measurement of platelet function have resulted in extremely variable findings, with a reported frequency of aspirin nonresponsiveness varying from 5% to 60%. Proposed explanations for the mechanism of aspirin resistance have included poor patient compliance, suboptimal aspirin absorption, in- creased platelet receptor activity, increased platelet cyclooxygenase-2 and isoprostane activity, and genetic polymorphisms of cyclooxygenase-1 or platelet glycoprotein IIIa.32

The reported prevalance of nonresponsiveness to clopidogrel has varied from 5% to 44%, primarily related to the loading dose (300 mg vs 600 mg) and the dosing protocol.33 Patient noncompliance, individual variability in the platelet ADP receptor, polymorphisms in the platelet ADP receptor, and polymorphisms in the cyto- chrome P450 drugs that metabolize clopidogrel, especially CYP2C19.

4. Discuss laboratory methods for the determination of patient responsiveness to aspirin and clopido- grel, including light transmission aggregometry, Plateletworks, PFA-100, and the Accumetrics Verify- Now assay.

Light transmission aggregometry (turbidimetric aggregometry) measures the in vitro response of platelets to various chemical agents (i.e., aggregating agents, platelet agonists) that induce platelet functional responses. In the clinical laboratory, platelet aggregometry is utilized for the diagnosis of inherited and acquired platelet disorders, the assay of von Willebrand factor activity (ristocetin cofactor assay), the diagnosis of heparin- induced thrombocytopenia, and monitoring patients receiving antiplatelet drugs.34 Conventional optical plate-

Contemporary Issues in Diagnostic Hemostasis and Thrombosis ASCP Annual Meeting, October 2011 Case #1 - Clinical Case #3 - Discussion 25

let aggregometers are modified spectrophotometers that measure light transmission through platelet-rich plasma (PRP). Although the turbidity of fresh PRP limits light transmission, transmission progressively in- creases as platelet aggregation causes the formation of larger and larger particles. More recent innovations of platelet aggregometry include whole blood aggregometers and lumiaggregometers. Whole blood aggre- gometers require less patient blood and provide faster turn-around time than optical aggregometers.35 Lumi- aggregometers simultaneously measure platelet aggregation and ATP secretion to provide a more accurate diagnosis of platelet function defects.36 The platelet agonists routinely used in the clinical laboratory to differ- entiate various platelet function defects include adenosine diphosphate (ADP), epinephrine, collagen, risto- cetin, and arachidonic acid. Other agonists, such as thrombin, vasopressin, serotonin, thromboxane A2 (TXA2), platelet activating factor, and other agents are used by research and specialized clinical laboratories. Conventional platelet aggregation is a complex laboratory assay that is particularly sensitive to the assay conditions, as well as drugs and other substances in the blood. Because of these influences, platelet aggre- gometry is an advanced, manually intense, costly assay restricted to specific clinical circumstances. A variety of commercial instruments and reagents for platelet aggregometry are available from Chrono-Log Corpora- tion (Havertown, PA), Bio/Data Corporation (Horsham, PA), and Helena Laboratories (Beaumont, TX).

Plateletworks (Helena Laboratories, Beaumont, Texas) is a rapid in vitro point of care platelet aggregation screening technique based on impedance platelet counting and specifically developed for cardiopulmonary bypass and cardiac catheterization settings.37, 38 The technique uses whole blood to measure the change in the platelet count due to platelet aggregation. Whole blood is drawn and separate 1 mL aliquots are placed into tubes containing EDTA (baseline), an ADP agonist, a collagen agonist, and an arachidonic acid agonist. The tubes are well mixed by inversion and the platelet count is measured in each tube using a hematology analyzer. The percent aggregation is calculated in each tube by the following formula:

Baseline Platelet Count - Agonist Platelet Count % Aggregation= x100 Baseline Platelet Count

The Plateletworks assay has been used to monitor the reversal of platelet inhibition with clopidogrel or NSAISs in elective cardiac surgery patients, monitoring the efficacy of therapy with platelet GpIIb-IIIa antago- nists in patients undergoing percutaneous coronary intervention or receiving medical therapy for non-ST ele- vation acute coronary syndromes, and predicting post-operative blooding and blood product utilization in pa- tients undergoing cardiac surgery with cardiopulmonary bypass.39-41

The PFA-100 (Siemens Healthcare Diagnostics, Deerfield, IL) is a rapid, automated laboratory instrument that is sensitive to quantitative and qualitative abnormalities of platelets and von Willebrand factor (vWF).42-44 In the PFA-100, citrated whole blood is aspirated from a reservoir under constant vacuum conditions through a microscopic 150 µm aperture. This aperture is cut into a biologically active nitrocellulose membrane in a disposable cartridge device coated with a combination of platelet agonists. These agonists are either colla- gen (fibrillar Type I equine tendon) and epinephrine (C/Epi) or collagen and adenosine-5’- diphosphate (C/ ADP). The blood is forced through the aperture at a high shear rate that roughly corresponds to the flow con- ditions present in small arteries. As the blood is forced through the aperture, platelets undergo adherence, activation and aggregation on the membrane surrounding the aperture and progressively form a plug that finally occludes the aperture. The closure time (CT) is the time required for the complete occlusion to occur. The PFA-100 is more rapid and less expensive than the bleeding time for the evaluation of platelet function. Since there is a good correlation between the bleeding time and the PFA-100 in certain patient populations, there is a trend to replace the bleeding time with the PFA-100 for a first-line screening test for platelet dys- function in patients undergoing preoperative evaluation. Other clinical applications of the PFA-100 include the following:

Contemporary Issues in Diagnostic Hemostasis and Thrombosis ASCP Annual Meeting, October 2011 Case #3 - Discussion 26

• Screening for impaired primary hemostasis, including von Willebrand disease and inherited platelet dysfunction • Verification of desmopressin acetate (DDAVP) therapy efficacy in pre-surgical patients • Evaluate platelet dysfunction due to antiplatelet drugs • Evaluate patients with high-risk pregnancy, menorrhagia, and other diseases for bleeding dysfunc- tion

There are several cavets in the clinical utilization of the PFA-100. Strict adherence to specimen requirements, specimen transportation, and specimen processing is required, since the PFA-100 is affected by critical pre- analytical variables such as hematocrit or platelet count, blood collection technique, and transportation through pneumatic tube systems.45 Since the PFA-100 has been reported as insensitive to some patients with platelet function defects, clinical correlation is critical, with follow-up with a different screening technique in cases of high clinical suspicion. The PFA-100 is insensitive to alterations in the quantity or quality of fi- brinogen and therefore has not been shown to be useful in evaluating patients for the presence of dysfibrino- genemia or hypofibrinogenemia. It is not sensitive to defects or deficiencies in the classic coagulation factors and appears to have little if any significant utility in assessing Hemophilia A and B.

The VerifyNow System (Accumetrics, San Diego, CA), formerly the Ultegra Accumetrics RPFA, is a point-of- care platelet function analyzer specifically designed and marketed for the monitoring of selected antiplatelet drugs, including aspirin, P2Y12 inhibitors (clopidogrel, prasugrel, ticlopidine), and GPIIb/IIIa antagonists.37, 46, 47 The VerifyNow System is comprised of: (1) a small analyzer that uses turbidimetric optical detection to measure the agglutination of fibrinogen-coated microparticles in anticoagulated whole blood, and (2) dispos- able, single-use assay cartridges that contain fibrinogen-coated beads, a platelet agonist, and other neces- sary reagents. The patient collection tube (whole blood in 2mL Greiner partial fill Vacuette with 3.2% sodium citrate) is placed into the analyzer where an appropriate aliquot is automatically dispensed from the into the assay cartridge without operator intervention. The results are available in several minutes. In the assay, platelets with unblocked receptors are activated and cause microparticle agglutination with a change in opti- cal light transmission.

The Accumetrics Verifynow test for aspirin resistance uses an instrument which measures the increase in light transmittance when platelets aggregate in response to arachidonic acid. The results of the test are ex- pressed in Aspirin Reaction Units (ARU), which reflect the rate and extent of platelet aggregation.48-53 The expected therapeutic range for platelet function in patients receiving aspirin is 350-549 ARU. Patients on as- pirin with greater than 550 ARU are considered aspirin resistant. After aspirin discontinuation, an ARU of less than 550 indicates residual responsiveness.

The VerifyNow P2Y12 Plavix Response Assay quantitates platelet inhibition after P2Y12 receptor blockade, including the thienopyridine class of drugs (i.e., clopidogrel, prasugrel and ticagrelor). The P2Y12 assay can be useful for detecting thienopyridine resistance, monitoring thienopyridine dosage effect, and ensuring pa- tient compliance. Patients with adequate platelet inhibition have an increased risk of bleeding, and discon- tinuation of thienopyridine administration is recommended five days prior to surgery. The results of the P2Y12-mediated platelet aggregation are reported in P2Y12 Reactivity Units (PRU). Baseline platelet func- tion (BASE) is measured in a separate channel and reported as BASE PRU Units. The percent change from baseline is calculated from the BASE and PRU results and reported as percent P2Y12 Inhibition (%).47, 54, 55

The target level for patients receiving chronic antiplatelet therapy is a percent P2Y12 inhibition ≥ 50%. Pa- tients with suboptimal platelet inhibition show a PRU > 235 U and/or percent P2Y12 Inhibition <20%. These patients may have an increased risk of cardiac events, such as stent thrombosis and recurrent cardiac events. Baseline PRU values below 193 U usually indicate an interfering substance (i.e., recent glycoprotein IIb/IIIa inhibitor use) and are not likely to be valid.

Contemporary Issues in Diagnostic Hemostasis and Thrombosis ASCP Annual Meeting, October 2011 Case #3 - References 27

The results of the VerifyNow assay should be interpreted in light of all the clinical and laboratory data avail- able on the patient. Strict adherence to the collection protocol and timing of analysis is essential for meaning- ful results, and the possibility of improper sample collection or handling should be considered for patients with unexpected P2Y12 assay results. The assay has not been studied in patients with significant thrombocy- topenia (PLT <100 x 10e9/L) and is contraindicated in patients with von Willebrand factor deficiency and in- herited platelet disorders, such as Glanzmann thrombasthenia and Bernard-Soulier syndrome.

5. Does Antonio have aspirin resistance, clopidogrel resistance, or both?

Antonio appears to be resistant to clopidogrel, since his P2Y12 inhibition is <20% (15%) and his PRU is > 235 U (252 U). The results of the assay appear valid, since the baseline is > 193 U (295 U).

References 12. Vane JR, Flower RJ, Botting RM. History of aspirin and its mechanism of action. Stroke. Dec 1990;21(12 1. Varon D, Spectre G. Antiplatelet agents. Hematology Suppl):IV12-23. Am Soc Hematol Educ Program. 2009:267-272. 13. Sneader W. The discovery of aspirin: a reappraisal. 2. Thijs T, Nuyttens BP, Deckmyn H, Broos K. Platelet BMJ. Dec 23-30 2000;321(7276):1591-1594. physiology and antiplatelet agents. Clin Chem Lab Med. Dec 2010;48 Suppl 1:S3-13. 14. Warner TD, Mitchell JA. Cyclooxygenase-3 (COX-3): filling in the gaps toward a COX continuum? Proc Natl 3. Hall R, Mazer CD. Antiplatelet drugs: a review of their Acad Sci U S A. Oct 15 2002;99(21):13371-13373. pharmacology and management in the perioperative pe- riod. Anesth Analg. Feb 2011;112(2):292-318. 15. Gibson PC. Aspirin in the treatment of vascular dis- eases. Lancet. Dec 24 1949;2(6591):1172-1174. 4. Steinhubl SR. Historical Observations on the Discovery of Platelets, Platelet Function Testing and the First Anti- 16. Craven LL. Experiences with aspirin (Acetylsalicylic platelet Agent. Curr Drug Targets. Jun 30 2011. acid) in the nonspecific prophylaxis of coronary thrombo- sis. Miss Valley Med J. Jan 1953;75(1):38-44. 5. Yousuf O, Bhatt DL. The evolution of antiplatelet therapy in cardiovascular disease. Nat Rev Cardiol. Jul 12 2011. 17. Botting RM. Vane's discovery of the mechanism of ac- tion of aspirin changed our understanding of its clinical 6. Stone WM, Naidu SG, Chapital A, Money SR. Antiplate- pharmacology. Pharmacol Rep. May-Jun let medications: old and emerging therapies. Ann Vasc 2010;62(3):518-525. Surg. Jan 2010;24(1):140-148. 18. Awtry EH, Loscalzo J. Aspirin. Circulation. Mar 14 7. Peter K, Myles PS. Perioperative antiplatelet therapy: A 2000;101(10):1206-1218. knife-edged choice between thrombosis and bleeding still based on consensus rather than evidence. Thromb Hae- 19. Vane JR, Botting RM. The mechanism of action of as- most. Mar 8 2011;105(4). pirin. Thromb Res. Jun 15 2003;110(5-6):255-258.

8. Patrono C, Rocca B. The future of antiplatelet therapy in 20. Gurbel PA, Tantry US. Clopidogrel resistance? Thromb cardiovascular disease. Annu Rev Med. 2010;61:49-61. Res. 2007;120(3):311-321.

9. Kei AA, Florentin M, Mikhailidis DP, Elisaf MS, Libero- 21. Savi P, Nurden P, Nurden AT, Levy-Toledano S, Her- poulos EN. Review: Antiplatelet drugs: what comes next? bert JM. Clopidogrel: a review of its mechanism of action. Clin Appl Thromb Hemost. Feb 2011;17(1):9-26. Platelets. 1998;9(3-4):251-255.

10. Lopes RD. Antiplatelet agents in cardiovascular dis- 22. de Labriolle A, Doazan JP, Lemesle G, Bonello L. ease. J Thromb Thrombolysis. Apr 2011;31(3):306-309. Genotypic and Phenotypic Assessment of Platelet Func- tion and Response to P2Y12 Antagonists. Curr Cardiol 11. Hayden J. Aspirin--a short review of its history, effects Rep. Jul 29 2011. and uses. Queens Nurs J. Sep 1975;18(6):172-174.

Contemporary Issues in Diagnostic Hemostasis and Thrombosis ASCP Annual Meeting, October 2011 Case #3 - References 28

23. Deeks ED. Ticagrelor: a review of its use in the man- 36. Cattaneo M. Light transmission aggregometry and agement of acute coronary syndromes. Drugs. May 7 ATP release for the diagnostic assessment of platelet 2011;71(7):909-933. function. Seminars in thrombosis and hemostasis. Mar 2009;35(2):158-167. 24. Cannon CP, Harrington RA, James S, et al. Compari- son of ticagrelor with clopidogrel in patients with a 37. Michelson AD. Methods for the measurement of plate- planned invasive strategy for acute coronary syndromes let function. The American journal of cardiology. Feb 2 (PLATO): a randomised double-blind study. Lancet. Jan 2009;103(3 Suppl):20A-26A. 23 2010;375(9711):283-293. 38. Campbell J, Ridgway H, Carville D. Plateletworks: a 25. De Servi S, Savonitto S. How to explain the reduced novel point of care platelet function screen. Molecular cardiovascular mortality in the ticagrelor arm of the diagnosis & therapy. 2008;12(4):253-258. PLATO trial? Int J Cardiol. Jun 2 2011;149(2):265-267. 39. White MM, Krishnan R, Kueter TJ, Jacoski MV, Jen- 26. Cambria-Kiely JA, Gandhi PJ. Possible mechanisms nings LK. The use of the point of care Helena ICHOR/ of aspirin resistance. J Thromb Thrombolysis. Feb Plateletworks and the Accumetrics Ultegra RPFA for as- 2002;13(1):49-56. sessment of platelet function with GPIIB-IIIa antagonists. Journal of thrombosis and thrombolysis. Dec 27. Bhatt DL. Aspirin resistance: more than just a labora- 2004;18(3):163-169. tory curiosity. J Am Coll Cardiol. Mar 17 2004;43(6):1127-1129. 40. Stegnar M. Platelet function tests and resistance to antiplatelet therapy. Srpski arhiv za celokupno lekarstvo. 28. Sweeny JM, Gorog DA, Fuster V. Antiplatelet drug Jan 2010;138 Suppl 1:59-63. 'resistance'. Part 1: mechanisms and clinical measure- ments. Nat Rev Cardiol. Apr 2009;6(4):273-282. 41. Craft RM, Chavez JJ, Snider CC, Muenchen RA, Car- roll RC. Comparison of modified Thrombelastograph and 29. Musallam KM, Charafeddine K, Bitar A, et al. Resis- Plateletworks whole blood assays to optical platelet ag- tance to aspirin and clopidogrel therapy. Int J Lab Hema- gregation for monitoring reversal of clopidogrel inhibition tol. Feb 2011;33(1):1-18. in elective surgery patients. The Journal of laboratory and 30. Kolandaivelu K, Bhatt DL. Overcoming 'resistance' to clinical medicine. Jun 2005;145(6):309-315. antiplatelet therapy: targeting the issue of nonadherence. Nat Rev Cardiol. Aug 2010;7(8):461-467. 42. Kundu SK, Heilmann EJ, Sio R, Garcia C, Davidson RM, Ostgaard RA. Description of an in vitro platelet func- 31. Collaboration AT. Collaborative meta-analysis of ran- tion analyzer--PFA-100. Semin Thromb Hemost. 1995;21 domised trials of antiplatelet therapy for prevention of Suppl 2:106-112. death, myocardial infarction, and stroke in high risk pa- tients. BMJ. Jan 12 2002;324(7329):71-86. 43. Mammen EF, Comp PC, Gosselin R, et al. PFA-100 system: a new method for assessment of platelet dys- 32. Martin CP, Talbert RL. Aspirin resistance: an evalua- function. Semin Thromb Hemost. 1998;24(2):195-202. tion of current evidence and measurement methods. Pharmacotherapy. Jul 2005;25(7):942-953. 44. Favaloro EJ. Clinical utility of the PFA-100. Seminars in thrombosis and hemostasis. Nov 2008;34(8):709-733. 33. Gurbel PA, Tantry US. Stent thrombosis: role of com- pliance and nonresponsiveness to antiplatelet therapy. 45. Hubner U, Bockel-Frohnhofer N, Hummel B, Geisel J. Reviews in cardiovascular medicine. 2007;8 Suppl The effect of a pneumatic tube transport system on plate- 1:S19-26. let aggregation using optical aggregometry and the PFA- 100. Clin Lab. 2010;56(1-2):59-64. 34. Hayward CP, Moffat KA, Raby A, et al. Development of North American consensus guidelines for medical labo- 46. Simon DI, Liu CB, Ganz P, et al. A comparative study ratories that perform and interpret platelet function testing of light transmission aggregometry and automated bed- using light transmission aggregometry. American journal side platelet function assays in patients undergoing per- of clinical pathology. Dec 2010;134(6):955-963. cutaneous coronary intervention and receiving abciximab, eptifibatide, or tirofiban. Catheter Cardiovasc Interv. Apr 35. McGlasson DL, Fritsma GA. Whole blood platelet ag- 2001;52(4):425-432. gregometry and platelet function testing. Seminars in thrombosis and hemostasis. Mar 2009;35(2):168-180.

Contemporary Issues in Diagnostic Hemostasis and Thrombosis ASCP Annual Meeting, October 2011 Case #3 - References 29

47. McLean DS, Cannon CP. A point-of-care assay to measure platelet aggregation in patients taking clopido- grel. Future Cardiol. May 2006;2(3):255-267.

48. Lordkipanidze M, Pharand C, Schampaert E, Turgeon J, Palisaitis DA, Diodati JG. A comparison of six major platelet function tests to determine the prevalence of aspi- rin resistance in patients with stable coronary artery dis- ease. Eur Heart J. Jul 2007;28(14):1702-1708.

49. Nielsen HL, Kristensen SD, Thygesen SS, et al. Aspi- rin response evaluated by the VerifyNow Aspirin System and light transmission aggregometry. Thromb Res. 2008;123(2):267-273.

50. van Werkum JW, Harmsze AM, Elsenberg EH, Bou- man HJ, ten Berg JM, Hackeng CM. The use of the Veri- fyNow system to monitor antiplatelet therapy: a review of the current evidence. Platelets. Nov 2008;19(7):479-488.

51. Blais N, Pharand C, Lordkipanidze M, Sia YK, Merhi Y, Diodati JG. Response to aspirin in healthy individuals. Cross-comparison of light transmission aggregometry, VerifyNow system, platelet count drop, thromboelastogra- phy (TEG) and urinary 11-dehydrothromboxane B(2). Thromb Haemost. Aug 2009;102(2):404-411.

52. Madsen EH, Saw J, Kristensen SR, Schmidt EB, Pit- tendreigh C, Maurer-Spurej E. Long-term aspirin and clopidogrel response evaluated by light transmission ag- gregometry, VerifyNow, and thrombelastography in pa- tients undergoing percutaneous coronary intervention. Clin Chem. May 2010;56(5):839-847.

53. Gremmel T, Steiner S, Seidinger D, Koppensteiner R, Panzer S, Kopp CW. Comparison of methods to evaluate aspirin-mediated platelet inhibition after percutaneous intervention with stent implantation. Platelets. 2011;22(3):188-195.

54. Bouman HJ, Parlak E, van Werkum JW, et al. Which platelet function test is suitable to monitor clopidogrel re- sponsiveness? A pharmacokinetic analysis on the active metabolite of clopidogrel. J Thromb Haemost. Mar 2010;8(3):482-488.

55. von Beckerath N, Sibbing D, Jawansky S, et al. As- sessment of platelet response to clopidogrel with multiple electrode aggregometry, the VerifyNow P2Y12 analyzer and platelet Vasodilator-Stimulated Phosphoprotein flow cytometry. Blood Coagul Fibrinolysis. Jan 2010;21(1):46-52.

Contemporary Issues in Diagnostic Hemostasis and Thrombosis ASCP Annual Meeting, October 2011 The Case of the Architect with Post-operative Thrombocytopenia Clinical History: Nearly early every home and business in Mason City, MO has a plaque somewhere inside with the inscription “Built by Biggs.” Situated on the outskirts of Mason City, the offices of the Biggs Construction Company were surrounded by an army of red and yellow trucks and construction equipment. 65-year-old Luther Bigg’s philosophy of building was “fast and frugal,” and his well-organized construction crews could build any- thing within budget nearly overnight. Luther continually bounced from administrative meetings to on-site super- vision, as he tried to be personally on hand for the impor- tant phases of every construction project. However, the hard work and long hours were taking their toll on Luther. One morning, while working on the new Mason City Art Center, he had severe chest pain and was rushed to Mason City Medical Center, where he underwent three- vessel coronary artery bypass surgery the same after- noon. Luther’s postoperative course was complicated by transient atrial fibrillation and volume overload, causing him to remain ventilator-dependent. Intravenous heparin was administered to prevent the possibility of thrombo- sis. Luther’s platelet count was 200x109/L preoperatively and 182x109/L on postoperative day 2 but fell to 70 x 109/L on the seventh postoperative day. Fortunately, there were no clinical signs of thrombosis, which was confirmed by arterial and venous Doppler ultrasonogra- phy of all four extremities. The on-call vascular surgery resident speculated about the possibility of disseminated intravascular coagulation (DIC) as the cause of the thrombocytopenia and decided to order blood cultures, since Luther had a slight fever. That evening, the chief resident was quite outraged, and immediately gave other orders. What did he order? Case #4

Running Title or Section Title Case #4 - Discussion 31

Questions

1. What is the most likely etiology of the thrombocytopenia in Mr. Biggs?

Until proven otherwise, thrombocytopenia occurring in a patient receiving heparin in any form should be consid- ered heparin-induced thrombocytopenia (HIT).1-7 In these patients, all available clinical and laboratory data should be reviewed to exclude other causes of acute thrombocytopenia, especially spurious thrombocytopenia, with disseminated intravascular coagulation (DIC) and drug-induced thrombocytopenia (DIT) caused by other medications the patient is receiving. Other than heparin, common causes of DIT include antibiotics (van- comycin, beta-lactam antibiotics), quinine, antiepileptic drugs (phenytoin, carbamazepine, valproic acid), glyco- protein IIb/IIIa inhibitors, etc. Overall, more than 300 drugs have been associated with thrombocytopenia.

Heparin can induce thrombocytopenia by two different mechanisms. A transient, non-immune mediated de- crease in platelets occurs in about 30% of patients after the initiation of heparin therapy, and was formerly re- ferred to as HIT type I, non-immune mediated HIT, or heparin-associated thrombocytopenia. This form of thrombocytopenia, caused by a direct interaction of heparin with the platelet surface, is usually mild (platelet count > 100 x 109/L), occurs within the first few days after heparin administration, and resolves within several days, even without heparin discontinuation. Type I HIT is not associated with an increased risk of thrombosis.

In contrast to HIT Type I, Type II HIT (hereafter referred to as HIT) is a dramatic and potentially catastrophic complication. This uncommon complication of heparin therapy occurs in approximately 1-3% of heparinized pa- tients, but is significantly higher in cardiovascular patients, who are often repeatedly exposed to heparin.8, 9 Thrombocytopenia can occur at any time, but typically begins 5 to 10 days after initial heparin exposure, or within hours of reexposure to heparin. Although more common in patients receiving unfractionated heparin than low molecular weight heparin (LMWH), HIT can occur with heparin exposure of any type, including subcutane- ous heparin injections or heparin flushes through intravenous lines or catheters. Unlike other types of DIT, HIT is associated with a low risk of bleeding, but a substantial risk of arterial or venous thrombosis. This thrombosis is unique in that it is typically a “white thrombus” composed predominantly of platelets and little fibrin. The over- all occurrence of thrombosis is approximately 20-50%, with the highest incidence in women and post-surgical patients. In medical inpatients, a high unfractionated heparin dose, exposure to heparin for more than five days were the most significant risk factors for the development of HIT.10 The thrombosis is usually of venous origin but arterial thrombosis can occur, particularly in patients with arteriosclerosis. There is a high incidence of mor- bidity and mortality in HIT patients with thrombosis, largely from acute myocardial infarction, pulmonary embo- lism, and peripheral arterial thrombosis. Other adverse consequences can result for microvascular thrombosis and include skin necrosis, adrenal hemorrhagic necrosis, and anaphylactoid reactions. The term heparin- induced thrombocytopenia and thrombosis (HITT) has been used for patients with thrombocytopenia and thrombosis. The 11 The financial impact of developing HIT are considerable, and largely a consequence of medication costs, prolonged in-hospital stay, diagnostic and therapeutic interventions, laboratory tests, blood transfusions.11, 12 In one study, the projected annual financial impact was estimated at $700,000 to $1,000,000 for an institution with an incidence of 50 HIT cases per year.

The etiology of HIT has been extensively studied in the laboratory, although many details have yet to be resolved.13 Heparin binds to platelet factor 4 (PF4, cytokine CXCL4) to form an antibody-inducing heparin/PF4 complex. PF4 is a 70 amino acid protein produced by megakaryocytes and released from the alpha-granules of activated platelets. The normal physiologic role of PF4 as an inflammatory mediator is not completely under- stood, but it has high binding affinity for negatively-charged glycosaminoglycans, including heparin, and may also bind to thrombomodulin to enhance protein C activation. Recently, the molecular structure of the heparin/ PF4 complex has been reported to mimic repetitive viral epitopes that are thought to trigger a rapid T-cell inde- pendent immune response with little anamnestic response.14 Regardless of their mechanism of formation, anti- PF4/heparin antibodies in HIT are primarily of the IgG2 class and interact with the heparin/PF4 complex to form large macromolecular immune complexes with high affinity for the FCγRIIA (CD32) receptors on platelets and monocytes. The activated platelets release additional PF4 that further exacerbate the platelet activation and lead to thrombin generation and thrombin generation (“thrombin storm”). Anti-heparin/PF4 antibodies have re-

Contemporary Issues in Diagnostic Hemostasis and Thrombosis ASCP Annual Meeting, October 2011 Case #4 - Discussion 32

cently been found to inhibit the protein C regulatory system, which also potentiates the prothrombotic state.15 Other investigators have reported a role for heparin contaminants in activating the contact factor system or acti- vated monocytes in promoting thrombous formation.16-18 The antibody-coated platelets are eventually removed by the reticuloendothelial system, leading to thrombocytopenia. The pathogenesis of the thrombosis is multifac- torial and less well defined. The major pathogenic mechanisms include the generation of procoagulant, platelet- derived microparticles and other procoagulants from activated platelets, thrombin generation resulting from heparin neutralization by PF4 released by activated platelets, and vascular damage resulting from the interac- tion of the anti-heparin/PF4 to heparan sulfate and other heparinoid-like structures on the endothelial cell.

The clinical diagnosis of HIT is complicated by the variability in the platelet count, the time course of the throm- bocytopenia, and the existence of other diseases and medication effects in the target patient population.19 Since the platelet count can remain in the normal range (i.e., > 150,000 x 109/L) in approximately 10% of patients with HIT, a relative decrease in the baseline or post-operative platelet count of > 50% is used as a diagnostic marker of the disease, rather than an absolute platelet count. Most patients with HIT have platelet counts in the range of 50,000 to 60,000 x 109/L, and a few have severe thrombocytopenia of <30 x 109/L. “Typical-onset” HIT de- velops 5-14 days after heparin exposure, while “rapid-onset” HIT occurs within 24 hours in patient previously exposed to heparin, usually within the past 30 days. Rare patients with “delayed-onset” thrombocytopenia de- velop HIT more than 10 days following heparin exposure. Because of the variability in the temporal onset of HIT, the American College of Chest Physicians (ACCP) recommends the procurement of a base-line platelet count prior to heparin administration, 24 hours after beginning therapy, and at regular intervals until the discontinua- tion of heparin therapy. Routine platelet count monitoring is recommended at least every other day in high-risk patients and every two to three days in moderate risk patients. Routine platelet count monitoring is usually not recommended for patients receiving LMWH.

Laboratory assays for HIT that are sensitive, specific, and rapidly available do not exist at this time. In order to facilitate rapid clinical decision-making, Warkentin and Heddle developed a clinical scoring system for HIT (4 T) based on four available clinical criteria, including the platelet count, time course for the development of throm- bocytopenia, the presence or absence of thrombosis, and the presence or absence of other causes of thrombocytopenia.20-25 Algorithms combing the 4 T scoring system with laboratory criteria were subsequently developed, including the HIT Expert Probability Score.26, 27 The 4 T clinical scoring system is detailed in Table VI. Table VI 4 T’s Clinical Scoring System for HIT*

Points (0, 1, or 2 for each category, maximum score = 8)

Parameter 0 1 2

Platelet count <30% fall or nadir <10 x 109/L 30-50% fall or nadir 10-19 x >50% fall and nadir >20 x 109/L 109/L

Timing of platelet count de- Fall < 4 days without recent Uncertain onset days 5-10 Clear onset days 5-10; or crease exposure (missing platelet counts), or onset ≦1 day in case of hepa- onset ≦1 day in case of hepa- rin exposure <30 days ago rin exposure 30-100 days ago

Thrombosis or other se- None Progressive or recurrent New thrombosis (confirmed); quelae thrombosis; non-necrotizing skin necrosis; acute systemic (erythematous) skin lesions; reaction after intravenous suspected thrombosis (not heparin bolus injection proven)

Other cause of thrombocy- Definite Possible None apparent topenia

Scoring - ≦3 points: Low probability for HIT; 4-5 points: Medium probability; 6-8 points: High probability *Warkentin and Heddle, 2003

Contemporary Issues in Diagnostic Hemostasis and Thrombosis Case #4 - Discussion 33

2. What immediate action should be undertaken?

The following actions must be undertaken in any patient suspected of having HIT, type II:

• All forms of heparin must be immediately discontinued. • Anticoagulation with a direct thrombin inhibitor (DTI) must be started and continued until the platelet count increases to >100 x 109/L. • The patient must be carefully monitored for the development of thromboembolic disease. • Anti-vitamin K (i.e., warfarin) therapy is initiated when the platelet count exceeds 100 x 109/L with a pe- riod of dual therapy “bridging” until a therapeutic INR is achieved

Treatment with a DTI is critical to decrease the risk of thrombosis after the discontinuation of heparin therapy.23, 28-30 If DTI therapy is not began, the risk of thrombosis is approximately 50% after twenty days. The first DTI, hirudin, was originally isolated from the saliva of the medicinal leech (Hirudo medicinalis) in the 1800‘s, but it was not widely used after the discovery of heparin in the early 1900‘s. Worldwide, five DTI’s are presently used in patient care. Four DTI’s are FDA-approved, and three are used for the treatment of HIT (Table II). DTI’s are classified as univalent or bivalent depending on their interaction with thrombin.31 Bivalent DTI’s (Lepirudin, Bivalirudin, and Desirudin) bind to both the active (catalytic) site and fibrinogen-binding, regulatory exosite (exosite 1) on the thrombin molecule, while univalent DTI’s (Argatroban, Dabigatran) bind selectively to the ac- tive site. Another univalent DTI, Ximelagatran (Exanta, Exarta) was withdrawn from the market after reports of hepatotoxicity. However, fondaparinux (Arixtra, GlaxoSmithKline, Brentford, Middlesex, United Kingdom), a synthetic pentasaccharide Factor Xa inhibitor, and other drugs are under evaluation.32

Lepirudin (Refludan, Bayer Corporation, West Haven, CT) is a desulfated recombinant form of hirudin ap- proved by the FDA in 1998 for the treatment of HIT. Lepirudin inhibits both free and bound and does not inter- act with HIT antibodies. However, it is largely excreted by the kidneys, so care must be used in renal patients with HIT. Bivalirudin (Angiomax, Angiox, The Medicines Company, Parsippany, NJ) is a short, synthetic com- pound comprised of the C-terminal and N-terminal portions of the active peptide regions of the hirudin mole- cule. It has a rapid onset and short half-life, inhibits both circulating and clot-bound thrombin, and inhibits thrombin-mediated platelet activation. Since lepirudin is largely eliminated by proteolytic enzymes, it can be used in patients with renal or hepatic dysfunction. It is used, in combination with aspirin and GPIIb/IIIa platelet inhibitors for undergoing percutaneous transluminal coronary angioplasty (PTCA) or percutaneous coronary intervention (PCI), and in patients at risk of HIT undergoing PCI. Desirudin (Revasc, Rhone-Poulenc-Rorer) is a synthetic, essentially irreversible recombinant form of huridin licensed in Europe for the prevention of DVT in patients undergoing elective hip and knee replacement surgery.

Argatroban (Arganova, Argatra, Novastan, GlaxoSmithKline, Brentford, Middlesex, United Kingdom) is small molecule, L-arginine derivative, and reversible thrombin inhibitor that binds to free, fibrin-bound, and clot- associated thrombin.33, 34 Argatroban was FDA-approved in 2000 for the prevention/treatment of thrombosis in HIT, and in 2002 for the treatment of patients at risk of HIT undergoing percutaneous coronary intervention (PCI).35 Argatroban undergoes hepatic clearance, and must be dose-adjusted in patients with hepatic dysfunc- tion.

Dabigatran etexilate (Pradaxa, Pradax, Prazaxa, Boehringer Ingelheim, Ingelheim Germany). Dabigatran is a derivative of a benzamidine-based thrombin inhibitor originally discovered in the 1980’s. Dabigatran is orally administered as a prodrug that is converted by hepatic esterases into the active metabolite. Dabigatran was approved by the FDA in 2010 for the for prevention of stroke in patients with non-valvular atrial fibrillation. The active metabolite is excreted by the kidneys, and it is contraindicated in patients with renal dysfunction.

Vitamin K antagonists such as warfarin are contraindicated for the treatment of patients with HIT since they rapidly decrease the production of protein C. , in the thrombotic environment of HIT, is a adverse factor that could trigger severe microvascular thrombosis and additional clinical consequences.

Contemporary Issues in Diagnostic Hemostasis and Thrombosis ASCP Annual Meeting, October 2011 Case #4 - Discussion 34

Table VII Features of Direct Thrombin Inhibitors Used to Treat HIT

Drug Lepirudin Argatroban Bivalirudin

Half-life 1.3 hours 39-51 minutes 25 minutes

Clearance Renal Hepatic Renal, proteolytic

Dose IV bolus: 0.4 mg/kg IV infusion: 2 µg/kg/min IV bolus: 0.75 mg/kg IV infusion: 0.15 mg/kg/h IV infusion: 1.75 mg/kg/h

Monitoring aPTT, ACT aPTT, ACT ACT, aPTT, thrombin time

FDA approved Yes Yes No

Mechanism of action Bivalent Univalent Bivalent

3. What additional laboratory assays should be requested? Would a positive laboratory result confirm the diagnosis of this disease entity in Mr. Biggs?

Laboratory assays for the detection of anti-heparin:PF4 complex antibodies should be obtained, although the results have a poor correlation with clinical symptomatology and are usually of little help in the immediate man- agement of the patient with suspected HIT. In this regard, the role of laboratory testing in HIT diagnosis was comprehensively reviewed by Baldwin and collaborators. Overall, they found anti-heparin/PF4 antibodies are discovered in approximately 17% of patients treated with UFH, and 8% of those receiving LMWH. However, only 20% of antibody-positive patients develop thrombocytopenia, and very few of those develop significant thrombosis.36

These assays include: (1) antibody assays that directly determine the presence of the antibody in plasma or serum by enzyme immunoassay (EIA) and other techniques, and (2) functional assays, which indirectly deter- mine the presence of the antibody by measuring platelet activation in the presence of heparin.37 The HIT EIA assays are highly sensitive (~ 99%), commonly available, and relatively easy and rapid to perform. Unfortu- nately, they are less specific (50-90%), and cannot differentiate pathogenic from non-pathogenic antibodies. The functional assays are much more specific, but technically difficult to perform and less readily available. Consequently, the recommended course of action in most laboratories is to screen for the presence of HIT an- tibodies by EIA, and to follow-up with a functional assay in positive patients where the diagnosis still remains controversial.

Conventional solid-phase EIA assays for HIT antibodies in patient serum utilize a PF4/polyanion complex (PF4:heparin or PF4:polyvinylsulfonate (PVS)) coated on microtiter trays as a capture antigen. Patient antibod- ies binding the the PF4/polyanion complex are detected with an enzyme-labeled (usually alkaline phosphatase) anti-human globulin detection antibody. The presence of an anti-heparin:PF4 antibody induces a chromogenic reaction with the release of a colored reaction product measured as a change in the optical density (OD). A cutoff (usually 0.4) is specified by each manufacturer to differentiate a positive and negative result, although some authorities believe a higher OD should be used to increase the specificity of the assay and avoid overdiagnosis.38-40 Since the antibody avidity determines the OD units and may correlate with the pathogenic significance of the antibody, some laboratories report the quantitative OD unit value.27, 38, 39, 41-43 In most sys- tems, “weak” antibodies have an OD value in the range of 0.4 to 1.0, while stronger antibodies produce an OD > 1.0. Unfortunately, some HIT EIA assays can detect the clinically irrelevant IgA and IgM antibodies, as well as the pathogenic IgG forms.44, 45 These assays may also yield false positive results in the presence on autoimmune-related antibodies, immune complexes, and other non-HIT related antibodies.46-48 False negative results can occur in patients with low-avidity, low titer antibodies. The diagnostic specificity of the assay can be

Contemporary Issues in Diagnostic Hemostasis and Thrombosis ASCP Annual Meeting, October 2011 Case #4 - Discussion 35

increased with a “confirmatory” step using high concentrations of heparin that disrupt formation of the reaction and/or IgG-specific immunoassays.49 Commercial HIT EIA assays are available from several companies (Asserachrom-HPIA, Stago, Asniere, France; GTI Diagnostics, Brookfield, WI; Aniara Hyphen BioMed, Mason, OH).50 Other HIT immunoassays have been reported but are not widely used in the clinical laboratory. These include the fluid-phase EIA and particle gel immunoassay.51

Table VIII Major Laboratory Assays for PF4/Heparin Antibodies

Assay Specimen/Principle Advantages Disadvantages

Serotonin Release Assay Heat-treated patient se- High sensitivity Uses radioisotopes (SRA) rum High specificity Technically demanding Donor platelets isolated, Not readily available labeled with 14C sero- tonin, washed incubated with patient serum in presence of heparin, 14C measured

Platelet aggregation Patient platelet-poor High specificity Low sensitivity plasma Technique dependent Platelet anti-PF4 antibod- Technically time- ies induces aggregation consuming and demand- of normal platelets in ing presence of heparin Not readily available

EIA EIA with PF4/heparin- High sensitivity Low specificity coated microplates and Relative technical sim- Poor correlation with SRA patient serum or plasma plicity

The 14C-serotonin release assay (platelet serotonin release assay, SRA), platelet aggregation test (PAT), and heparin-induced platelet activation (HIPA) test are the most widely used functional assays for the detection of anti-heparin:PF4 antibodies. The SRA, developed by Sheridan et al. in 1986, uses FcγRIIa receptor- phenotyped “normal” platelets from patients reactive to heparin:PF4 antibodies.52 The platelets are incubated with radioactive serotonin, washed, resuspended in calcium-containing assay buffer. and then incubated with the heat-treated test serum in the presence and absence of low, therapeutic (0.1 to 0.3 U/mL) and high, su- pratherapeutic (10 to 100 U/mL) concentrations of heparin. Platelet activation is indicated by the release of ra- dioactive serotonin into the incubation supernatant, with a positive result signified by the release of > 20% of the radioactive serotonin to therapeutic levels of heparin, but not to supratherapeutic levels. Supratherapeutic heparin concentrations should decrease serotonin release by altering the binding ratio and decreasing the for- mation of the IgG/PF4/heparin immune complexes. Platelet activation at both low and high heparin concentra- tions (“intermediate” result) indicate the presence of preformed immune complexes or non-heparin dependent antibodies.53 Fc(gamma)RIIa receptor blocking monoclonal antibodies can also be used to confirm the specific- ity of the analysis. In view of the high specificity of the SRA, it is commonly used to confirm the diagnosis of HIT in patient’s with a positive HIT ELISA assay. Some procedural variations further increase the sensitivity by However, in some patients, non-drug-dependent antiplatelet autoantibodies can induce a false positive result. Due to the expense and technical complexity of the SRA, it is only performed by a few laboratories in the United States. The HIPA is similar to the SRA, but uses the macroscopic agglutination of platelets in a micro- titer tray as the end point. Non-radioactive variations of the SRA measure serotonin release by EIA or platelet- derived microparticles by flow cytometry.54, 55 The source of the control platelets is an important factor, since platelet FcγRIIa receptor polymorphisms at amino acid 131 influence the binding of heparin-dependent antibod- ies. In this regard, control platelets with histidine-histidine (His-His) or histidine-arginine (His-Arg) at FcγRIIa-

Contemporary Issues in Diagnostic Hemostasis and Thrombosis ASCP Annual Meeting, October 2011 Case #4 - Discussion 36

131 were found to greatly increase the sensitivity of the assay relative to platelets with arginine-arginine (Arg-Arg).56 The platelet FcγRIIa phenotype may also influence the risk of developing HIT. (Brandt, 1995)

The heparin platelet aggregation test (HPAT) uses a commercial platelet aggregometer to quantitatively meas- ure the aggregation of donor platelets (citrated platelet rich-plasma or washed platelets) by patient platelet-poor plasma in the presence of low-dose (0.2-0.5 U/mL) and high-dose (100 U/mL) unfractionated heparin.57 The formation of antibody/PF4/heparin immune complexes activate the platelet FcγRIIa receptor and cause platelet aggregation. The assay is considered positive with a platelet aggregation response of > 20% with low dose heparin, and <20% aggregation with high-dose heparin. The specificity of the HPAT is higher than that of the EIA, but the sensitivity is lower (50-80%). The assay requires meticulous attention to detail, with the concentra- tion of heparin and the source of the control platelets being the major determinants of the sensitivity and speci- ficity of the reaction. In this regard, it is best to use a mixture of platelets from several HIT antibody responsive donors. A related assay, the heparin-induced platelet aggregation (HIPA) assay uses washed normal platelets, patient serum, and heparin in microtiter trays, with a visual assessment of platelet aggregation. The lag time until platelet aggregation occurs is inversely proportional to the strength of the reaction. 58, 59 Lumiaggregome- try, with measurement of ADP release, has also been utilized.

The flow cytometric detection of heparin-induced thrombocytopenia has been reported by the measurement of platelet activation using platelet microparticles detection, annexin V binding to phosphatidylserine, and expres- sion of the platelet activation marker P-selectin (CD62p).60-62

4. Who discovered heparin? What is the chemical structure of heparin? How is heparin manufactured?

Dr. William Henry Howell, former dean of the School of Medicine at Johns Hopkins University and chairman of the Department of Physiology reoriented his research studies to the increasingly important field of coagulation in 1916. As part of this effort, Dr. Howell enlisted the help of Jay McLean, a medical student transferring from the University of California, to isolate procoagulant substances from animal tissue. Dr. Howell was initially skeptical when McLean claimed to have isolated a fat-soluble coagulation-inhibiting compound, or “anticoagu- lant,” from canine hepatic tissue but later repeated McLean’s work and isolated a water-soluble polysaccharide anticoagulant that he named “heparin,” from the ancient Greek word (hepar, liver).63 In 1920 Howell The struc- ture of heparin was elucidated by Erik Jorpes at Karolinska Institute in 1936, a technique for producing a safe salt solution was identified by Dr. Charles Best at the University of Toronto, and human clinical trials began in Toronto in 1935.64-68 The experimental and clinical work of Dr. Gordon Murray, a prominent Toronto heart sur- geon, was crucial for the perfection of heparin as a human anticoagulant.

Arterial thrombosis was reported in patients receiving heparin in the late 1950’s and early 1960’s, but an asso- ciation with thrombocytopenia was never made because platelet counts were not routinely monitored at that time. However, in the early 1970’s a group of vascular surgeons at Duke University finally associated thrombo- sis with heparin administration, and proved that it was an immune-mediated problem.69 Diagnostic assays for HIT were developed in the 1980’s, and therapeutic agents other than aspirin and warfarin developed in the 1990’s.

Heparin is a glycosaminoglycan polymer (3 kDa to 30 kDa) of variably-sulfated, highly negatively-charged, re- peating disaccharide units, most commonly 2-O-sulfated iduronic acid and 6-O-sulfated, N-sulfated gluco- samine, IdoA(2S)-GlcNS(6S).70 Heparin is naturally produced by basophils and mast cells, stored in their se- cretory granules, and released into the vasculature at the sites of tissue injury. The normal biological function of heparin is unclear; it may act as an anticoagulant, but may also have a role in host defense against bacteria and other foreign substances. Pharmaceutical-grade heparin is isolated from animal tissues, usually bovine lung or porcine intestinal mucosa, through a complex extraction process.

Contemporary Issues in Diagnostic Hemostasis and Thrombosis ASCP Annual Meeting, October 2011 Case #4 - Discussion 37

5. What physiologic effects are exerted by heparin?

The anticoagulant effect of heparin is mediated by its interaction with anti-thrombin (AT). The binding of a spe- cific pentasaccharide sequence in the heparin polymer to the AT molecule results in a 1000-fold increase in AT reactivity against thrombin, factor Xa, and other proteases. Although the anti-factor Xa activity of unfractionated heparin requires only the presence of the pentasaccharide heparin sequence, the inactivation of thrombin is size dependent, and requires the binding of thrombin both to the pentasaccharide sequence and a second site proximal to the pentasaccharide sequence. A recent variety of fractionated, low-molecular weight heparins (LMWH) have been produced, as well as a synthetic analogue mirroring the pentascaaharide sequence (fon- daparinux). These newer drugs exhibit a more subtle anticoagulant effect than unfractionated heparin, since they target only factor Xa.

6. Describe the administration and pharmacokinetics of heparin.

UFH is not absorbed from the gastrointestinal tract because of its high negative charge density and large size. This property, and its short, but variable biologic half-life has precluded its application as an outpatient drug. However, UFH is the most widely used anticoagulant for hospitalized patients, to whom it is administered by intravenous (IV) or subcutaneous (SQ) routes. In the bloodstream, heparin is extensively bound to plasma pro- teins, von Willebrand factor, macrophages, and endothelial cells. IV heparin exerts a nearly immediate antico- agulant effect, but there is a delay of > 20 minutes after the subcutaneous administration of heparin. Heparin has an unusual non-linear does-response relationship, such that there is a disproportional increase in the half- life and intensity of the anticoagulant effect as the dosage is increased.

250

200

150 (Seconds)

100 aPTT

50

0 0.0 0.1 0.2 0.3 0.4 0.5 0.6 0.7 0.8 0.9 1.0 Heparin (U/mL) (Anti-Xa Chromogenic Assay)

Fig 2. A typical heparin dose-response curve. The solid line represents the regression line of aPTT values plot- ted against heparin levels determined in fresh plasma specimens from at least 60 patients on heparin therapy by the anti-factor Xa assay. In this case, the therapeutic aPTT range corresponding to heparin levels of 0.3 to 0.7 U/mL is approximately 75-125 seconds. Contemporary Issues in Diagnostic Hemostasis and Thrombosis ASCP Annual Meeting, October 2011 Case #4 - Discussion 38

Heparin is cleared from the body by dual mechanisms, including a rapid, saturable phase mediated by the re- ticuloendothelial system, and a slower, non-saturable, dose-independent phase due to renal clearance.

5. What is the most common complication of heparin therapy?

Hemorrhage is the most common complication of postoperative heparin therapy. The risk of hemorrhage is small in anti-coagulated patients without underlying complications, and often presents as hematuria or mild oozing from a mucosal site. However, life-threatening bleeding from the brain, gastrointestinal tract, and other sites can also occur. In general, the more prolonged the aPTT, the greater the likelihood of hemorrhage. Hepa- rin therapy may also aggravate preexisting minor hemorrhage from pathologic lesions, such as colonic carci- noma or gastric ulcer. For this reason, bleeding from the gastrointestinal, respiratory, or urinary tract in patients on heparin should prompt a thorough search for a potential source of the bleeding. The risk of bleeding varies with the dosage and duration of treatment. The overall risk of bleeding is approximately 10% overall, but as high as 20% with high-dose heparin therapy in certain patients, including elderly patients, and those with other illness, renal failure, alcohol abuse, etc.

Patients on long-term heparin therapy are at risk of developing other complications, including osteoporosis and spontaneous vertebral fractures.

6. How is heparin therapy monitored by the laboratory?

Traditionally, the aPTT has been used to monitor the therapeutic effect of heparin. This test remains the most readily available and practical method to follow anti-coagulation by heparin. However the relationship between the aPTT and the concentration of heparin varies between labs reflective of differences in reagents, method- ologies, sample size, and other factors. The current recommendations are for each lab to standardize the aPTT therapeutic range to an anti-factor Xa assay of 0.3 to 0.7 U/mL or a direct heparin measurement by protamine titration of 0.2 to 0.4 U/mL. The current standardized recommended therapeutic ranges of aPTT are reported with each test result and are regularly updated for each new reagent lot, or other changes in laboratory rea- gents or instrumentation. One heparin unit (the "Howell Unit") is the quantity of heparin required to keep 1 mL of cat's blood fluid for 24 hours at 0 °C. This is approximately equivalent to 0.002 mg of pure heparin.

The “Brill-Edwards” protocol is the most widely used technique to measure the heparin-responsiveness of aPTT assay in a clinical laboratory. In the Brill-Edwards protocol, 50-100 plasma samples from different indi- viduals receiving unfractionated heparin are obtained. The heparin concentration of each sample is determined by the chromogenic anti-Factor Xa technique, and the corresponding aPTT of each sample is measured with the laboratories reagents and instrumentation. A heparin response curve is constructed by plotting the aPTT values against the corresponding heparin concentrations (Fig. 2). Regression analysis is used to determine the range of aPTT values corresponding to UFH levels of 0.3-0.7 IU/mL (“heparin therapeutic range”). Theoreti- cally, heparin dosing to maintain aPTT values within the therapeutic range will prevent over- or underdosing.

A “baseline” aPTT should be obtained prior to the administration of heparin. Regular monitoring should begin with the administration of heparin and continue regularly thereafter. Due to the complicated pharmacodynamics of heparin, and the potential for changes in clearance, the aPTT should be monitored daily, even after a thera- peutic dose is found. Monitoring becomes a more difficult problem in patients with an abnormal aPTT at base- line, those who require higher than average doses of heparin, and those who are undergoing transition to oral anticoagulation. Citrated blood for measurement of the aPTT should be drawn six hours after a heparin bolus, and the result should be used to adjust the next dose.

Heparin monitoring by measurement of heparin plasma levels using the anti-factor-Xa assay is highly recom- mended in any patient with a known preexisting condition that may alter the heparin concentration-aPTT rela- tionship or produce heparin resistance.29, 71-77 In view of the many disadvantages of the aPTT, there is a grow- ing consensus that the anti-factor Xa assay should be used for monitoring all patients receiving UFH. Disad- vantages of the aPTT include the lack of a relationship between the aPTT and the clinical efficacy of heparin, Contemporary Issues in Diagnostic Hemostasis and Thrombosis ASCP Annual Meeting, October 2011 Case #4 - References 39

variation in the responsiveness of commercial aPTT reagents and reagent/instrument combinations to heparin, the nonlinearity of the heparin dose response curve, and the lack of sensitivity of the aPTT to drugs such as LMWH and factor Xa inhibitors.

Treatment options for patients resistant to heparin include increasing the dose of heparin, using LMWH, a di- rect thrombin inhibitor, fondaparinux, fresh frozen plasma (FFP), and/or AT concentrate.

Final diagnosis: Heparin-induced thrombocytopenia (HIT)

inpatients. British journal of haematology. Aug References: 2011;154(3):373-377.

1. Ortel TL. Heparin-induced thrombocytopenia: when a low 11. Baroletti S, Piovella C, Fanikos J, Labreche M, Lin J, platelet count is a mandate for anticoagulation. Hematology Goldhaber SZ. Heparin-induced thrombocytopenia (HIT): / the Education Program of the American Society of clinical and economic outcomes. Thrombosis and Hematology American Society of Hematology Education haemostasis. Dec 2008;100(6):1130-1135. Program. 2009:225-232. 12. Wilke T, Tesch S, Scholz A, Kohlmann T, Greinacher A. 2. Arepally GM, Ortel TL. Heparin-induced The costs of heparin-induced thrombocytopenia: a thrombocytopenia. Annual review of medicine. patient-based cost of illness analysis. Journal of thrombosis 2010;61:77-90. and haemostasis : JTH. May 2009;7(5):766-773.

3. Butt A, Aronow WS, Chandy D. Heparin-induced 13. Sandset PM. Immunobiology of heparin-induced thrombocytopenia and thrombosis. Comprehensive therapy. thrombocytopenia. Current topics in microbiology and 2010;36:23-27. immunology. 2010;341:193-202.

4. Hong MS, Amanullah AM. Heparin-induced 14. Greinacher A. Heparin-induced thrombocytopenia. thrombocytopenia: a practical review. Reviews in Journal of thrombosis and haemostasis : JTH. Jul 2009;7 cardiovascular medicine. Winter 2010;11(1):13-25. Suppl 1:9-12.

5. Otis SA, Zehnder JL. Heparin-induced thrombocytopenia: 15. Kowalska MA, Krishnaswamy S, Rauova L, et al. current status and diagnostic challenges. American journal Antibodies associated with heparin-induced of hematology. Sep 2010;85(9):700-706. thrombocytopenia (HIT) inhibit activated protein C generation: new insights into the prothrombotic nature of 6. Rondina MT, Walker A, Pendleton RC. Drug-induced HIT. Blood. Jul 19 2011. thrombocytopenia for the hospitalist physician with a focus on heparin-induced thrombocytopenia. Hospital practice. Apr 16. Thachil J. Heparin induced thrombocytopenia with 2010;38(2):19-28. thrombosis: a two step process? Hematology. Jun 2008;13(3):181-182. 7. Cuker A. Heparin-induced thrombocytopenia: present and future. Journal of thrombosis and thrombolysis. Apr 17. Qian Y, Pan J, Zhou X, Weiser P, Lu H, Zhang L. 2011;31(3):353-366. Molecular mechanism underlines heparin-induced thrombocytopenia and thrombosis. Progress in molecular 8. Demma LJ, Levy JH. Diagnosing heparin-induced biology and translational science. 2010;93:395-421. thrombocytopenia in cardiac surgical patients: not as easy as you think. Anesthesia and analgesia. Apr 18. Rauova L, Hirsch JD, Greene TK, et al. Monocyte-bound 2011;112(4):747-749. PF4 in the pathogenesis of heparin-induced thrombocytopenia. Blood. Dec 2 2010;116(23):5021-5031. 9. Yoon JH, Jang IK. Heparin-induced thrombocytopenia in cardiovascular patients: pathophysiology, diagnosis, and 19. Greinacher A, Levy JH. HIT happens: diagnosing and treatment. Cardiology in review. May-Jun evaluating the patient with heparin-induced 2011;19(3):143-153. thrombocytopenia. Anesthesia and analgesia. Aug 2008;107(2):356-358. 10. Kato S, Takahashi K, Ayabe K, et al. Heparin-induced thrombocytopenia: analysis of risk factors in medical

Contemporary Issues in Diagnostic Hemostasis and Thrombosis ASCP Annual Meeting, October 2011 Case #4 - References 40

20. Crowther MA, Cook DJ, Albert M, et al. The 4Ts scoring low-molecular-weight heparin era for anticoagulant therapy? system for heparin-induced thrombocytopenia in Thromb Haemost. Nov 2009;102(5):892-899. medical-surgical intensive care unit patients. Journal of critical care. Jun 2010;25(2):287-293. 32. Badger NO. Fondaparinux (Arixtra(R)), a safe alternative for the treatment of patients with heparin-induced 21. Gruel Y, Regina S, Pouplard C. Usefulness of pretest thrombocytopenia? Journal of pharmacy practice. Jun clinical score (4Ts) combined with immunoassay for the 2010;23(3):235-238. diagnosis of heparin-induced thrombocytopenia. Current opinion in pulmonary medicine. Sep 2008;14(5):397-402. 33. Dhillon S. Argatroban: a review of its use in the management of heparin-induced thrombocytopenia. 22. Khan MM, Noble DW, Watson HG. Heparin-induced American journal of cardiovascular drugs : drugs, devices, thrombocytopenia and the utility of the 4Ts score in the and other interventions. 2009;9(4):261-282. Intensive Care Unit. British journal of haematology. Mar 2011;152(5):672-674. 34. Sparks ML. Argatroban therapy in heparin-induced thrombocytopenia. AACN advanced critical care. Jan-Mar 23. Lassila R, Antovic JP, Armstrong E, et al. Practical 2009;20(1):37-43. viewpoints on the diagnosis and management of heparin-induced thrombocytopenia. Seminars in thrombosis 35. Babuin L, Pengo V. Argatroban in the management of and hemostasis. Apr 2011;37(3):328-336. heparin-induced thrombocytopenia. Vascular health and risk management. 2010;6:813-819. 24. Pouplard C, Gueret P, Fouassier M, et al. Prospective evaluation of the '4Ts' score and particle gel immunoassay 36. Baldwin ZK, Spitzer AL, Ng VL, Harken AH. specific to heparin/PF4 for the diagnosis of heparin-induced Contemporary standards for the diagnosis and treatment of thrombocytopenia. Journal of thrombosis and haemostasis : heparin-induced thrombocytopenia (HIT). Surgery. Mar JTH. Jul 2007;5(7):1373-1379. 2008;143(3):305-312.

25. Strutt JK, Mackey JE, Johnson SM, Sylvia LM. 37. Walenga JM, Jeske WP, Fasanella AR, Wood JJ, Assessment of the 4Ts pretest clinical scoring system as a Bakhos M. Laboratory tests for the diagnosis of predictor of heparin-induced thrombocytopenia. heparin-induced thrombocytopenia. Semin Thromb Hemost. Pharmacotherapy. Feb 2011;31(2):138-145. 1999;25 Suppl 1:43-49.

26. Cuker A, Arepally G, Crowther MA, et al. The HIT Expert 38. Warkentin TE, Sheppard JI, Moore JC, Sigouin CS, Probability (HEP) Score: a novel pre-test probability model Kelton JG. Quantitative interpretation of optical density for heparin-induced thrombocytopenia based on broad measurements using PF4-dependent expert opinion. J Thromb Haemost. Dec enzyme-immunoassays. J Thromb Haemost. Aug 2010;8(12):2642-2650. 2008;6(8):1304-1312.

27. Ruf KM, Bensadoun ES, Davis GA, Flynn JD, Lewis DA. 39. Weiss BM, Shumway NM, Howard RS, Ketchum LK, A clinical-laboratory algorithm incorporating optical density Reid TJ. Optical density values correlate with the clinical value to predict heparin-induced thrombocytopenia. probability of heparin induced thrombocytopenia. Journal of Thrombosis and haemostasis. Mar 2011;105(3):553-559. thrombosis and thrombolysis. Dec 2008;26(3):243-247.

28. Hook KM, Abrams CS. Treatment options in 40. Berry C, Tcherniantchouk O, Ley EJ, et al. heparin-induced thrombocytopenia. Current opinion in Overdiagnosis of heparin-induced thrombocytopenia in hematology. Sep 2010;17(5):424-431. surgical ICU patients. Journal of the American College of Surgeons. Jul 2011;213(1):10-17. 29. Prechel M, Jeske WP, Walenga JM. Laboratory methods and management of patients with heparin-induced 41. Kim SY, Kim HK, Han KS, et al. Utility of ELISA optical thrombocytopenia. Methods in molecular biology. density values and clinical scores for the diagnosis of and 2010;663:133-156. thrombosis prediction in heparin-induced thrombocytopenia. The Korean journal of laboratory medicine. Jan 30. Warkentin TE. Agents for the treatment of 2011;31(1):1-8. heparin-induced thrombocytopenia. Hematology/oncology clinics of North America. Aug 2010;24(4):755-775, ix. 42. Whitlatch NL, Perry SL, Ortel TL. Anti-heparin/platelet factor 4 antibody optical density values and the confirmatory 31. Laux V, Perzborn E, Heitmeier S, et al. Direct inhibitors procedure in the diagnosis of heparin-induced of coagulation proteins - the end of the heparin and thrombocytopenia. Thrombosis and haemostasis. Oct 2008;100(4):678-684.

Contemporary Issues in Diagnostic Hemostasis and Thrombosis ASCP Annual Meeting, October 2011 Case #4 - References 41 53. Moore JC, Arnold DM, Warkentin TE, 43. Zwicker JI, Uhl L, Huang WY, Shaz BH, Bauer KA. Warkentin AE, Kelton JG. An algorithm for resolving Thrombosis and ELISA optical density values in hospitalized 'indeterminate' test results in the platelet serotonin release patients with heparin-induced thrombocytopenia. J Thromb assay for investigation of heparin-induced Haemost. Dec 2004;2(12):2133-2137. thrombocytopenia. Journal of thrombosis and haemostasis : JTH. Sep 2008;6(9):1595-1597. 44. Arepally G, Reynolds C, Tomaski A, et al. Comparison of PF4/heparin ELISA assay with the 14C-serotonin release 54. Gobbi G, Mirandola P, Tazzari PL, et al. Flow cytometry assay in the diagnosis of heparin-induced detection of serotonin content and release in resting and thrombocytopenia. Am J Clin Pathol. Dec activated platelets. Br J Haematol. Jun 1995;104(6):648-654. 2003;121(6):892-896.

45. Morel-Kopp MC, Aboud M, Tan CW, Kulathilake C, Ward 55. Fouassier M, Bourgerette E, Libert F, Pouplard C, C. Heparin-induced thrombocytopenia: evaluation of IgG Marques-Verdier A. Determination of serotonin release from and IgGAM ELISA assays. International journal of laboratory platelets by HPLC and ELISA in the diagnosis of hematology. Jun 2011;33(3):245-250. heparin-induced thrombocytopenia: comparison with reference method by [C]-serotonin release assay. J Thromb 46. Alpert DR, Salmon JE. False-positive tests for Haemost. May 2006;4(5):1136-1139. heparin-induced thrombocytopenia in patients with antiphospholipid syndrome and systemic 56. Bachelot-Loza C, Saffroy R, Lasne D, Chatellier G, erythematosus: a rebuttal. Journal of thrombosis and Aiach M, Rendu F. Importance of the haemostasis : JTH. Jun 2010;8(6):1439-1441. FcgammaRIIa-Arg/His-131 polymorphism in heparin-induced thrombocytopenia diagnosis. Thromb 47. Pauzner R, Greinacher A, Selleng K, Althaus K, Haemost. Mar 1998;79(3):523-528. Shenkman B, Seligsohn U. False-positive tests for heparin-induced thrombocytopenia in patients with 57. Chong BH, Burgess J, Ismail F. The clinical usefulness antiphospholipid syndrome and systemic lupus of the platelet aggregation test for the diagnosis of erythematosus. Journal of thrombosis and haemostasis : heparin-induced thrombocytopenia. Thromb Haemost. Apr 1 JTH. Jul 2009;7(7):1070-1074. 1993;69(4):344-350.

48. Pouplard C, Leroux D, Regina S, Rollin J, Gruel Y. 58. Greinacher A, Michels I, Kiefel V, Mueller-Eckhardt C. A Effectiveness of a new immunoassay for the diagnosis of rapid and sensitive test for diagnosing heparin-associated heparin-induced thrombocytopenia and improved specificity thrombocytopenia. Thromb Haemost. Dec 2 when detecting IgG antibodies. Thrombosis and 1991;66(6):734-736. haemostasis. Jan 2010;103(1):145-150. 59. Look KA, Sahud M, Flaherty S, Zehnder JL. 49. Bakchoul T, Giptner A, Bein G, Santoso S, Sachs UJ. Heparin-induced platelet aggregation vs platelet factor 4 Performance characteristics of two commercially available enzyme-linked immunosorbent assay in the diagnosis of IgG-specific immunoassays in the assessment of heparin-induced thrombocytopenia-thrombosis. Am J Clin heparin-induced thrombocytopenia (HIT). Thrombosis Pathol. Jul 1997;108(1):78-82. research. Apr 2011;127(4):345-348. 60. Lee DH, Warkentin TE, Denomme GA, Hayward CP, 50. Izban KF, Lietz HW, Hoppensteadt DA, et al. Kelton JG. A diagnostic test for heparin-induced Comparison of two PF4/heparin ELISA assays for the thrombocytopenia: detection of platelet microparticles using laboratory diagnosis of heparin-induced thrombocytopenia. flow cytometry. Br J Haematol. Dec 1996;95(4):724-731. Semin Thromb Hemost. 1999;25 Suppl 1:51-56. 61. Prechel MM, Escalante V, Drenth AF, Walenga JM. A 51. Bryant A, Low J, Austin S, Joseph JE. Timely diagnosis colorimetric, metabolic dye reduction assay detects highly and management of heparin-induced thrombocytopenia in a activated platelets: application in the diagnosis of frequent request, low incidence single centre using clinical heparin-induced thrombocytopenia. Platelets. Jul 8 2011. 4T's score and particle gel immunoassay. Br J Haematol. Dec 2008;143(5):721-726. 62. Tomer A, Masalunga C, Abshire TC. Determination of heparin-induced thrombocytopenia: a rapid flow cytometric 52. Sheridan D, Carter C, Kelton JG. A diagnostic test for assay for direct demonstration of antibody-mediated platelet heparin-induced thrombocytopenia. Blood. Jan activation. Am J Hematol. May 1999;61(1):53-61. 1986;67(1):27-30.

Contemporary Issues in Diagnostic Hemostasis and Thrombosis ASCP Annual Meeting, October 2011 Case #4 - References 42

63. Copeland CE, Six CK. A tale of two anticoagulants: 76. Rosenberg AF, Zumberg M, Taylor L, LeClaire A, Harris warfarin and heparin. J Surg Educ. May-Jun N. The use of anti-Xa assay to monitor intravenous 2009;66(3):176-181. unfractionated heparin therapy. J Pharm Pract. Jun 2010;23(3):210-216. 64. Fye WB. Heparin: the contributions of William Henry Howell. Circulation. Jun 1984;69(6):1198-1203. 77. Tripodi A. Measuring the anticoagulant effect of direct factor Xa inhibitors. Is the anti-Xa assay preferable to 65. Kelton JG, Warkentin TE. Heparin-induced the prothrombin time test? Thromb Haemost. Apr thrombocytopenia: a historical perspective. Blood. Oct 1 2011;105(4):735-736. 2008;112(7):2607-2616.

66. Marcum JA. William Henry Howell and Jay McLean: the experimental context for the discovery of heparin. Perspect Biol Med. Winter 1990;33(2):214-230.

67. Messmore HL, Jr. Heparin-induced thrombocytopenia: historical review. Clinical and applied thrombosis/hemostasis : official journal of the International Academy of Clinical and Applied Thrombosis/Hemostasis. Oct 1999;5 Suppl 1:S2-6.

68. Wardrop D, Keeling D. The story of the discovery of heparin and warfarin. Br J Haematol. Jun 2008;141(6):757-763.

69. Rhodes GR, Dixon RH, Silver D. Heparin induced thrombocytopenia with thrombotic and hemorrhagic manifestations. Surgery, gynecology & obstetrics. Mar 1973;136(3):409-416.

70. Gandhi NS, Mancera RL. The structure of glycosaminoglycans and their interactions with proteins. Chem Biol Drug Des. Dec 2008;72(6):455-482.

71. Barrett YC, Wang Z, Frost C, Shenker A. Clinical laboratory measurement of direct factor Xa inhibitors: anti-Xa assay is preferable to prothrombin time assay. Thromb Haemost. Dec 2010;104(6):1263-1271.

72. Cuker A, Ptashkin B, Konkle BA, et al. Interlaboratory agreement in the monitoring of unfractionated heparin using the anti-factor Xa-correlated activated partial thromboplastin time. J Thromb Haemost. Jan 2009;7(1):80-86.

73. Favaloro EJ, Bonar R, Aboud M, et al. How useful is the monitoring of (low molecular weight) heparin therapy by anti-Xa assay? A laboratory perspective. Lab Hematol. 2005;11(3):157-162.

74. Hammerstingl C, Omran H, Tripp C, Poetzsch B. How useful is determination of anti-factor Xa activity to guide bridging therapy with enoxaparin? A pilot study. Thromb Haemost. Feb 2009;101(2):325-332.

75. Ignjatovic V, Summerhayes R, Gan A, et al. Monitoring Unfractionated Heparin (UFH) therapy: which Anti-Factor Xa assay is appropriate? Thromb Res. 2007;120(3):347-351.

Contemporary Issues in Diagnostic Hemostasis and Thrombosis ASCP Annual Meeting, October 2011 The Case of the Retired Realtor with Bruising and Headaches Clinical History: Homes are difficult to find in the Mason City, MO region. Moving vans continually enter the area, and most leave empty. Many Mason City families living in a home are deeply indebted to Sandra Branson, owner of the There’s No Place Like Home Real Estate Company. Sandra is comfortably wealthy and has a beautiful home in the exclusive Foxwood Forest Estates subdivision of Mason City. Sandra enjoys the excitement of finding another house for sale, and closing yet another deal, but she also wants to retire and spend more time with her family. Fate intervenes when Sandra notices several large while showing a home. The bruis- ing continues for several weeks. Sandra reluctantly goes to her family physician. He finds that the bruises are un- provoked, present mostly on the extremities, and seem to extravasate under the skin rather than surfacing. San- dra also complains of intermittent headaches and nu- merous blood blisters in her mouth. Sandra is immedi- ately referred to the Midwest Hemostasis Institute for de- finitive evaluation.

Dr. Marcus Carrington sees Sandra the next day. He finds that she has a long history of arthritis and a com- plex, but more recent history of bleeding related prob- lems. Her history began eight years previously for pro- fuse bleeding following a mastoidectomy. On that occa- sion, the her platelet count was normal, but her bleeding time was markedly increased (>20 minutes), and platelet aggregation studies revealed an absent platelet re- sponse to ADP. Other laboratory parameters were unre- markable. Five years previously she was seen for the evaluation of bleeding from the right ear. The PT, aPTT, platelet count, and fibrinogen were normal at that time, Case #5 Case #5 - Discussion 44

but the bleeding time was again prolonged to >20 minutes. Sandra was advised against the use of alcohol, as- pirin, and other drugs that could affect platelet function. She was seen two years later for petechiae and a bleed in the conjunctiva of the lower lid of her right eye. A tourniquet test on the right arm was strongly positive. One month later she developed massive bleeding in the right knee two days after an injection of Cortisone for pain.

LABORATORY STUDIES: Initial laboratory studies at the Midwest Hemostasis Institute reveal the following: Hgb - 8 g/dL, Hct - 28%, platelet count 28,000/µL. Peripheral blood smear examination reveals approximately 10% nu- cleated red blood cells and a rare "blast-like" cell. Serum B12, folate and ferritin levels were within normal limits.

Questions

1. What process could account for Sandra’s history of bleeding problems? What additional laboratory studies should be performed?

A bone marrow study was performed. The bone marrow was hypercellular (95% cellular) with 6% myelo- blasts. Cytogenetic analysis of the bone marrow revealed 47,XX,+8. Platelet aggregation studies were re- markable for a markedly decreased response to ADP and for a slight prolongation in the lag time for collagen of uncertain cause.

2. What disease process does Sandra have?

A diagnosis of refractory anemia with excess blasts (RAEB) was rendered.

3. What is the pathophysiologic explanation for the bleeding?

This is the unusual case of a patient with a (RAEB) that was manifested years earlier as a platelet function abnormality with intermittent, unusual episodes of bleeding.

The myelodysplastic syndromes (MDS) are a heterogenous group of chronic neoplasms characterized by variable peripheral blood cytopenias resulting from ineffective hematopoiesis.1-6 First identified as a distinct disease category in 1976, MDS is now recognized as one of the most common hematologic diseases. They are largely diseases of advanced age, but some cases occur in children and younger individuals. The inci- dence of MDs is approximately 35,000 to 55,000 cases per year in the United States, with 86% occurring in patients > 60 years of age.7, 8 Immunosuppression, prior chemotherapy or radiotherapy, and exposure to benzene and other solvents have been identified as risk factors for MDS. The molecular pathogenesis of MDS is not well understood, but is believed to be similar to acute myelogenous leukemia, with at least two somatic gene mutations leading to enhanced proliferation and impaired differentiation of hematologic pro- genitor cells.2, 9-12 Chromosomal abnormalities are common in MDS and include trisomy 8 and deletions of chromosomes 5, 7, 13, 20, and the sex chromosomes. Although the precise etiologic mechanism of many of these abnormalities remains to be discovered, the 5q- syndrome, associated with the loss of 5q31-33, has been associated with gene RPS14.13, 14 A variety of genetic mutations, including JAK2, TET2, and NPM1, have recently been identified in patients with MDS, and their significance is under investigation.13, 15-21

The clinical symptoms of MDS result from cytopenia, and include variable combinations of anemia, neutro- penia, and thrombocytopenia.2 Physical examination may reveal evidence of bleeding (petechiae, ecchymo- ses, epistaxis, , hematuria, melena), anemia (, tachycardia, or congestive heart failure), or infection (fever, cough, dysuria, shock).

MDS must be differentiated from other causes of anemia, neutropenia, and thrombocytopenia. This requires complete blood counts and peripheral blood smear examination, bone marrow examination with cytogenetic and molecular studies, and the exclusion of other diseases, including congenital diseases, vitamin deficien- cies, lupus, hepatitis, HIV, hemolytic anemia, plasma cell neoplasms, etc.).22 The bone marrow is usually hypercellular for the age of the patient, but may be hypocellular. However, the morphologic hallmark of MDS is aberrant morphologic changes in the erythroid, myeloid, and megakaryocytic cell lines.16, 23 Dysplastic

Contemporary Issues in Diagnostic Hemostasis and Thrombosis ASCP Annual Meeting, October 2011 Case #5 - Discussion 45

granulocytes in the bone marrow or peripheral blood may be hypersegmented or hyposegmented, and show decreased cytoplasmic granules (hypogranularity) or large, pseudo-Chediak Higashi granules. The periph- eral blood erythrocytes show anisopoikilocytosis, while erythroid precursors in the bone marrow may show binucleation, internuclear bridging, or aberrant nuclear features (irregularity, budding, strings).24 Peripheral blood platelets may be large, giant, and/or hypogranular, while megakaryocytes my be hyposegmented or hypersegmented 25. A Prussian blue stain for iron can reveal abnormal, iron-containing erythroid precursors (ringed sideroblasts). The World Health Organization (WHO) recently issued a revised and updated classifi- cation for MDS, summarized in Table IX.

Table IX WHO (2008) Classification of Myelodysplastic Syndromes

Disease Peripheral Blood Findings Bone Marrow Findings

Refractory cytopenia with unilineage Cytopenia, blasts <1%, monocytes Dysplasia (>10%) in one lineage dysplasia (RA, RN, RT) ≤1 x 109/L only, <5% blasts, <15% ringed sid- eroblasts

Refractory anemia with ringed sid- Anemia, no blasts, monocytes ≤1 X Erythroid dysplasia only, <5% blasts, eroblasts 109/L ≥15% ring sideroblasts

Refractory cytopenia with multiline- Cytopenia(s), <1% blasts, mono- Dysplasia in >10% of the cells of ≥2 age dysplasia cytes ≤1 X 109/L, no Auer rods myeloid lineages, <5% blasts, <15% ringed sideroblasts, no Auer rods

RAEB-1 Cytopenias, <5% blasts, monocytes Unilineage or multilineage dysplasia, ≤1 X 109/L, no Auer rods 5%-9% blasts, no Auer rods

RAEB-2 Cytopenias, 5%-19% blasts, mono- Unilineage or multilineage dysplasia; cytes ≤1,000/µL, Auer rods ± 10%-19% blasts; Auer rods ±

MDS with isolated del(5q) Anemia, normal to slightly increased Normal to increased megakaryo- platelets, <5% blasts, no Auer rods cytes with hypolobated nuclei, <5% blasts, del (5q) as sole cytogenetic abnormality, no Auer rods

An increased bleeding tendency is common in patients with a myelodysplastic syndrome. The bleeding is usually due to thrombocytopenia, but dysplastic megakaryopoiesis with morphologically and functionally aberrant platelets, thrombotic thrombocytopenic purpura, acquired factor VIII inhibitors, and other abnormalaities are sometimes found. 26-29 Platelet ultrastructural examination in these patients has revealed various abnormalities, including reduced, fused, or giant granules (storage pool deficiency); dilated canaliculi, and aberrant microtubules.30-32 Abnormal platelet membrane glycoprotein expression has also been described. A platelet function abnormality should be expected when the clinical bleeding manifestations are unusually severe for the degree of thrombocytopenia.

Final Diagnosis: Myelodysplastic syndrome with thrombocytopenia and acquired platelet function de- fect.

Contemporary Issues in Diagnostic Hemostasis and Thrombosis ASCP Annual Meeting, October 2011 Case #5 - References 46

References: 13. Jadersten M, Hellstrom-Lindberg E. New clues to the molecular pathogenesis of myelodysplastic syn- 1. Nimer SD. Myelodysplastic syndromes. Blood. May dromes. Exp Cell Res. May 1 2010;316(8):1390-1396. 15 2008;111(10):4841-4851. 14. Naresh KN, Reid A. Bone marrow morphology in 2. Orazi A, Czader MB. Myelodysplastic syndromes. myelodysplastic syndrome associated with isolated Am J Clin Pathol. Aug 2009;132(2):290-305. del(5q). Am J Hematol. Aug 2010;85(8):609-610.

3. Tefferi A, Vardiman JW. Myelodysplastic syn- 15. Hellstrom-Lindberg E. Significance of JAK2 and dromes. N Engl J Med. Nov 5 TET2 mutations in myelodysplastic syndromes. Blood 2009;361(19):1872-1885. Rev. Mar 2010;24(2):83-90.

4. Nguyen PL. The myelodysplastic syndromes. He- 16. Bacher U, Haferlach T, Kern W, Weiss T, Schnitt- matol Oncol Clin North Am. Aug 2009;23(4):675-691. ger S, Haferlach C. The impact of cytomorphology, cytogenetics, molecular genetics, and immunopheno- 5. Pilo F, Di Tucci AA, Dessalvi P, Caddori A, Ange- typing in a comprehensive diagnostic workup of lucci E. The evolving clinical scenario of myelodys- myelodysplastic syndromes. Cancer. Oct 1 plastic syndrome: the need for a complete and up to 2009;115(19):4524-4532. date upfront diagnostic assessment. Eur J Intern Med. Dec 2010;21(6):490-495. 17. Olsen RJ, Dunphy CH, O'Malley DP, Rice L, Ew- ton AA, Chang CC. The implication of identifying JAK2 6. Greenberg PL, Attar E, Bennett JM, et al. Myelo- ( V617F ) in myeloproliferative neoplasms and myelo- dysplastic syndromes. J Natl Compr Canc Netw. Jan dysplastic syndromes with bone marrow fibrosis. J 2011;9(1):30-56. Hematop. Sep 2008;1(2):111-117.

7. Rollison DE, Howlader N, Smith MT, et al. Epide- 18. Rocquain J, Carbuccia N, Trouplin V, et al. Com- miology of myelodysplastic syndromes and chronic bined mutations of ASXL1, CBL, FLT3, IDH1, IDH2, myeloproliferative disorders in the United States, JAK2, KRAS, NPM1, NRAS, RUNX1, TET2 and WT1 2001-2004, using data from the NAACCR and SEER genes in myelodysplastic syndromes and acute mye- programs. Blood. Jul 1 2008;112(1):45-52. loid . BMC Cancer. 2010;10:401.

8. Sekeres MA. The epidemiology of myelodysplastic 19. Schnittger S, Bacher U, Haferlach C, et al. Char- syndromes. Hematol Oncol Clin North Am. Apr acterization of NPM1-mutated AML with a history of 2010;24(2):287-294. myelodysplastic syndromes or myeloproliferative neoplasms. Leukemia. Jan 14 2011. 9. Epling-Burnette PK, List AF. Advancements in the molecular pathogenesis of myelodysplastic syndrome. 20. Sokol L, Caceres G, Rocha K, Stockero KJ, De- Curr Opin Hematol. Mar 2009;16(2):70-76. wald DW, List AF. JAK2(V617F) mutation in myelo- dysplastic syndrome (MDS) with del(5q) arises in ge- 10. Steensma DP. The spectrum of molecular aberra- netically discordant clones. Leuk Res. Jun tions in myelodysplastic syndromes: in the shadow of 2010;34(6):821-823. . Haematologica. Jun 2007;92(6):723-727. 21. Valent P, Wieser R. Update on genetic and mo- lecular markers associated with myelodysplastic syn- 11. Davids MS, Steensma DP. The molecular patho- dromes. Leuk Lymphoma. Mar 2009;50(3):341-348. genesis of myelodysplastic syndromes. Cancer Biol Ther. Aug 2010;10(4):309-319. 22. Mufti GJ, Bennett JM, Goasguen J, et al. Diagno- sis and classification of myelodysplastic syndrome: 12. Nimer SD. An update on the molecular patho- International Working Group on Morphology of myelo- genesis of myelodysplastic syndromes. Biol Blood dysplastic syndrome (IWGM-MDS) consensus pro- Marrow Transplant. Jan 2011;17(1 Suppl):S11-14. posals for the definition and enumeration of myelo-

Contemporary Issues in Diagnostic Hemostasis and Thrombosis ASCP Annual Meeting, October 2011 Case #5 - References 47 blasts and ring sideroblasts. Haematologica. Nov 32. Woessner S. Transmission electron microscopy in 2008;93(11):1712-1717. hematological diagnosis. Ultrastruct Pathol. May-Aug 2005;29(3-4):237-268. 23. Orazi A. Histopathology in the diagnosis and clas- sification of acute myeloid leukemia, myelodysplastic syndromes, and myelodysplastic/myeloproliferative diseases. Pathobiology. 2007;74(2):97-114.

24. Majumder D, Banerjee D, Chandra S, Banerjee S, Chakrabarti A. Red cell morphology in leukemia, hy- poplastic anemia and myelodysplastic syndrome. Pa- thophysiology. Dec 2006;13(4):217-225.

25. Germing U, Platzbecker U, Giagounidis A, Aul C. Platelet morphology, platelet mass, platelet count and prognosis in patients with myelodysplastic syn- dromes. Br J Haematol. Aug 2007;138(3):399-400.

26. Girtovitis FI, Ntaios G, Papadopoulos A, Ioannidis G, Makris PE. Defective platelet aggregation in myelodysplastic syndromes. Acta Haematol. 2007;118(2):117-122.

27. Shen YM, Frenkel EP. Acquired platelet dysfunc- tion. Hematol Oncol Clin North Am. Aug 2007;21(4):647-661, vi.

28. Sugimori N, Kondo Y, Shibayama M, et al. Aber- rant increase in the immature platelet fraction in pa- tients with myelodysplastic syndrome: a marker of karyotypic abnormalities associated with poor prog- nosis. Eur J Haematol. Jan 2009;82(1):54-60.

29. Mezzano D, Quiroga T, Pereira J. The level of laboratory testing required for diagnosis or exclusion of a platelet function disorder using platelet aggrega- tion and secretion assays. Semin Thromb Hemost. Mar 2009;35(2):242-254.

30. Cohen AM, Alexandrova S, Bessler H, Mittelman M, Cycowitz Z, Djaldetti M. Ultrastructural observa- tions on bone marrow cells of 26 patients with myelo- dysplastic syndromes. Leuk Lymphoma. Sep 1997;27(1-2):165-172.

31. Pamphilon DH, Aparicio SR, Roberts BE, Menys VC, Tate G, Davies JA. The myelodysplastic syndromes--a study of haemostatic function and platelet ultrastructure. Scand J Haematol. Nov 1984;33(5):486-491.

Contemporary Issues in Diagnostic Hemostasis and Thrombosis ASCP Annual Meeting, October 2011 Appendix #1 - Coagulation System Diagram 48

HK PK XII XIIa Contact - System XI XIa

Ca++ VIII PL IX IXa Intrinsic Pathway VIIIa

VII/VIIa Extrinsic PL Pathway TF Va X Xa Ca++ V Common Pathway Prothrombin Thrombin XIII

Fibrin X-linking Fibrinogen Fibrin

Phospholipid XIIIa (PLTs)

X-Linked Fibrin

Contemporary Issues in Diagnostic Hemostasis and Thrombosis ASCP Annual Meeting, October 2011 Riley’s Hemostasis Tables

Major Coagulation Factors

Other Molecular Plasma Minimal Major Vitamin K Factor Names Weight Concentra- Hemostasis Surgery Half-Life Dependent Function Major Activity (Daltons) tion (%) (%)

150-400 mg/ Fibrin Forms fibrin clot after cleavage by throm- I Fibrinogen 333,000 50-100 mg/dL >100 mg/dL 3-6 Day No dL precursor bin, fibrin clot stabilized by factor XIII

Thrombin precursor, activated by Serine II Prothrombin 72,000 100 µg/mL 10-20% 30-40% 50-80 Hr Yes prothrombinase complex on surface of protease activated platelets

Cell membrane glycoprotein on subendo- Tissue factor, thromboplastin, Tissue pro- thelial cells, platelets, monocytes, cofac- III 47,000 N/A N/A N/A N/A Cofactor CD142 tein tor for factor VII, member of cytokine re- ceptor class II family.

Essential component of the tenase and IV Calcium - 9-10.5 mg/dL N/A N/A N/A N/A Cofactor prothrombinase complexes

Proaccelerin, labile factor, Protein cofactor in the activation of V accelerator (Ac-) globulin 300,000 7-10 µg/mL 5-15% 25% 4.5-36 Hr. No Cofactor prothrombin by factor Xa; activated by thrombin VI FVa, Accelerin - Proconvertin, serum prothrom- Endopeptidase with gla residues; acti- Serine 2+ VII bin conversion accelerator 50,000 0.5-1 µg/mL 5-10% 10-20% 2-5 Hr Yes vated by thrombin in presence of Ca (SPCA), cothromboplastin protease

Antihemophiliac factor A, Activated by thrombin; cofactor in activa- VIII-AHF 293,000 0.2 µg/mL 1-5% 50-60% 8-12 Hr No Cofactor antihemophilic globulin (AHG) tion of factor X by factor IXa

Christmas Factor, antihemo- Endopeptidase with gla residues; acti- Serine IX philic factor B, plasma throm- 56,000 4-5 mg/mL 10-20% 50-60% 18-24 Hr Yes vated by factor XIa in presence of Ca2+ protease boplastin component (PTC) Endopeptidase with gla residues; acti- Stuart-Prower Factor Serine vated on surface of activated platelets by X 56,000 10 µg/mL 5-10% 15-20% 20-42 Hr Yes protease tenase complex and by factor VIIa in presence of tissue factor and Ca2+

XI Plasma thromboplastin antece- 160,00 2-7 µg/mL 10-15% 15-30% 40-80 Hr No Serine Endopeptidase; activated by factor XIIa dent protease (PTA) Endopeptidase; binds to exposed colla- Hageman Factor XII 76,000 30-40 µg/mL <10% 10% 50 Hr No Transpeptidase gen at site of vessel wall injury, activated by high-MW kininogen and kallikrein Protransglutaminase, fibrinoli- Transpeptidase; activated by thrombin in Serine XIII gase, fibrin stabilizing factor 320,000 30 µg/mL 1% 5% 7-14 Day No presence of Ca2+; stabilizes fibrin clot by protease (FSF) covalent cross-linking Single chain gamma globulin; role in Serine Prekallikrein PK, Fletcher factor 86,000 40 N/A N/A ? No early stage contact factor activtion, ki- protease nin formation, and fibrinolysis HMWK, Contact activation co- Source of kinin; links prekallikrein to High Molecular factor, negative surface for activation by sur- Weight Kinino- Fitzgerald, Flaujeac Williams 120,000 100 N/A N/A ? No Cofactor face bound factor XIIa; forms complex gen factor with factor XI and accelerates activation

Contemporary Issues in Diagnostic Hemostasis and Thrombosis ASCP Annual Meeting, October 2011 Variable multimers of 2050 amino acid Binding to (260 kDa) monomer; protects factor VIII von Willebrand 260,000 - vWF Variable N/A N/A 12 hours No proteins and from degradation in circulation; links col- Factor >10,000,000 cells lagen and GPIb; binds to many sub- stances

Cleaves plasma Cleaves vWF at the 842-Tyr-|-Met-843 in vWFCP; ADAMTS-13; ADAM and platelet vWF-Cleaving Pro- the A2 domain of the vWF subunit; TTP metallopeptidase with thrombo- 190,000 1 µg/mL N/A N/A ? No surface vWF tease results from ADAMTs-13 deficiency or spondin type 1 motif, 13 multimers into anti-ADAMTS-13 antibody smaller forms

Regulators/Inhibitors of Coagulation

Other Molecular Plasma Vitamin K Factor Names Weight Concentration Half-Life Dependent Function Major Activity

AT, antithrombin III, AT III, serpin Serine prote- Inhibits thrombin, factor Xa, factor IXa, factor Antithrombin peptidase inhibitor, serine prote- 58,000 0.12 mg/mL 3 days No inase inhibitor VIIa, factor XIa, and factor XIIa; activity inase inhibitor greatly enhanced by heparin

Activated (activated protein C, APC, protein Autoprothrombin IIA Serine Ca) by thrombin bound to thrombomodulin; Protein C Anticoagulant protein C 62,000 25 mg/L 8-12 Hr Yes protease inactivates factors Va and VIIIa in the pres- Blood coagulation factor XIV ence of calcium ions and phospholipid

Protein S - 84,000 25 mg/L ? Yes Cofactor Cofactor of protein degradation of coagula- tion factors Va and VIIIa. Type I mem- Thrombomodulin TM, CD141, BDCA-3, fetomodulin 74,000 Membrane N/A No brane protein, Forms complex with thrombin that activates protein endothelial protein C cell receptor

Circulates in association with plasma VLDL Tissue factor Endopepti- TFPI, serpin A10 34,000-40,000 75-120 ng/mL ? No lipoproteins, reversibly inhibits factor Xa; pathway inhibitor dase inhibitor FVIIa-TF complex is inhibited by TFPI-Xa complex Binds thrombin and promotes its association Cofactor for with phospholipid vesicles, reduces rate of Protein Z - 62,000 1400 ng/mL ? Yes ZPI coagulation factor Xa inhibition by anti- thrombin, enhances inhibition of factor Xa mediated by ZPI

Protein Z-Related ZPI, serpin peptidase inhibitor, Serine prote- Inhibits factor Xa activity in the presence of Protease Inhibitor PZ-dependent protease inhibitor, 72,000 4 ug/mL ? No ase inhibitor protein Z, calcium and phospholipid, inhibits factor XIa

Heparin cofactor A, antithrombin Serine prote- Inhibits thrombin; increased activity in pres- Heparin Cofactor II BM, dermatan sulfate cofactor, 65,600 90 ug/mL ? No ase inhibitor ence of heparin human leuserpin-2

Contemporary Issues in Diagnostic Hemostasis and Thrombosis ASCP Annual Meeting, October 2011 Components of Fibrinolytic System

Factor Other Molecular Plasma Half-Life Vitamin K Function Major Activity Names Weight Concentration Dependent

Plasmin proen- Activated by tPA, uPA, thrombin, fibrin, and Plasminogen Profibrionolysin 92,000 200 ug/mL 0.8-2.2 No zyme, serine prote- factor XII; degrades fibrin and other plasma Days ase proteins

Tissue-type plasmi- t-PA, tPA, PLAT, Alteplase 5 ug/mL (total) 5 minutes Plasminogen acti- Largely circulates in complex with PAI-1; nogen activator Reteplase 68,000 1 ug/mL (free) ? No vator converts plasminogen to plasmin

Activated by vitronectin; principal inhibitor PA!-1 PLANH1, PAI1, Endothelial 45,000 5-10 ng/mL 1 hour No Serine protease in- of tissue plasminogen activator (tPA) and plasminogen activator inhibitor hibitor urokinase (uPA)

Major inhibitor of circulating serine prote- Alpha-2 plasmin inhibitor, pri- ases, including plasmin, trypsin, and chy- Alpha 2- Serine protease in- motrypsin; cross-linked to fibrin by factor antiplasmin mary plasmin inhibitor or anti- 58,700 70 ug/mL ? No hibitor plasmin XIIIa; inhibits plasmin by binding irreversi- bly to active catalytic site, inhibits binding of plasmin to fibrin TAFI, plasma carboxypepti- dase B, pCPB, carboxypepti- Thrombin- dase U, CPU, carboxypepti- Fibrinolysis inhibi- Activated by thrombin-thrombomodulin ; activatable fibri- dase B-like protein, thrombin- 60,000 2.5 ug/mL ? ? tor down-regulates fibrinolysis by decreasing nolysis inhibitor activable fibrinolysis inhibitor, fibrin binding for t-PA and plasminogen thrombin-activatable fibrinoly- sis inhibitor

Special Platelet Components

Factor Other Molecular Plasma Source/Localization Function Major Activity Names Weight concentration

35,800, homotetra- Biologic function un- Heparin-binding protein, plasma marker of Beta-Thromboglobulin β-TG mer of 8800 Mr 18 ng/mL Platelet (alpha granule) known platelet activation subunit

Neutralizes anticoagulant activity of hepa- rin, stimulates histamine release from ba- Platelet Factor 4 PF4 29,000 6 ng/mL Platelet (alpha granule) Heparin neutralization sophils, neutrophil/monocyte chemotactic activity

Platelet adhesion and aggregation, inhibi- Platelet (alpha granule), Adhesive protein (cell- tion of angiogenesis, activation of apopto- many other cells. Inte- Thrombospondin TSP-1, thrombin-sensitive protein 450,000 440 ng/mL cell, cell-substrate), sis, immune regulation, and activation of gral membrane compo- multiple functions TGF-beta. Multiple receptors, including nent in some cells CD36, CD47, integrins

Roger S. Riley, M.D. Ph.D. VCU School of Medicine

Contemporary Issues in Diagnostic Hemostasis and Thrombosis ASCP Annual Meeting, October 2011